SlideShare uma empresa Scribd logo
1 de 205
Cardiology Lecture to IM
Board Exam oriented
MCQS
Picture Quiz
Explanatory notes

Dr Ihab Suliman
October 2013
Outline




MCQS
Picture Quiz
Explanatory notes
Old/Prior MI
Introductory MCQs








The best BB associated with significant &
proven Mortality benefit post MI ?
a- Atenolol
b- Propranolol
c- Timolol
d- carvedilol (Dilatrend)
e-Metoprolol tartrate, Short acting


d- carvedilol (Dilatrend)



Other Medications with mortality benefit post
MI are
Metoprolol succinate , bisoprolol ( Concor),
ACEI, ARBs , Spironolactone, statins


Q 23








A 56 year old obese female presents for a routine physical
examination. Her lipid profile  reveals a significantly elevated
triglyceride level of 355 mg/dL. Which of the following
medications can act to lower her triglyceride level by stimulating
the synthesis of lipoprotein lipase?
A) gemfibrozil
B) rosuvastatin
C) cholestyramine
D) ezetimibe
E) ketoconazole
A 23


A) gemfibrozil



Gemfibrozil acts by stimulating the synthesis of lipoprotein lipase to
degrade triglycerides  into fatty acids increasing their metabolism and
lowering blood levels. Elevated triglyceride levels can lead to atherosclerosis
and coronary artery disease.
Rosuvastatin (B) is an HMG-CoA reductase inhibitor which can cause
rhabdomyolysis or hepatic dysfunction (elevation in AST and ALT levels).
Cholestyramine (C) is a bile acid binding resin used to treat elevated lowdensity lipoprotein levels (LDL). Ezetimibe (D) is also used to treat elevated
LDL levels and acts by inhibiting cholesterol absorption at the brush border
of the small intestine. Ketoconazole (E), an anti-fungal medication,
significantly reduces LDL levels as well. 


Q1


A 19 year old boy with no past medical history passes out while
running at a school evnt. He states that he was feeling a little
dizzy prior to the event, but does not complains of chest pain or
palpitations. His blood pressure is 120/85, heart rate 85/min,
Respiratory Rate 12/min, and he is afebrile. His physical
examination reveals normal lung sounds, a II/VI mid-sytolic
creshendo-decreshndo murmur is heard at the right upper sternal
border which increases in intensity with Valsalva, an S4 heart
sound is also present. Laboratory studies are normal. What is the
most appropriate treatment at this time?






A) Start a Diuretic
 B) Permanent Pacemaker Implantation 
C) AICD Implantation 
D) Observation
 E) Start  Beta Blocker
A1


C) AICD Implantation




The commonest cardiovascular manifestation of
the Coxsackie infective agent is
a) cardiac tamponade
b) congenital Pericardial Defects
c) acute viral Pericarditis
d) pericardial Cysts
e) bronchogenic carcinoma


c) acute viral Pericarditis




35 year old obese women develops painful leg
on the 10th post operative day following
emergency surgery for a ruptured ectopic
pregnancy. She is likely to have ?
a) thromboangitis obliterans 
b) ruptured baker`s cyst 
c) deep vein thrombosis of the lower limb 
d) aortoiliac artery artherosclerosis 
e) embolus



c) deep vein thrombosis of the lower limb 
Specially if she is homozygous or heterozygous
for coagulopathy factor ?




A 56 year old gardener complains of sudden
pain with swelling behind the right knee. He also
complains of associated pain and swelling in the
calf. On examination a crescentic hematoma is
noted below the medial malleolus of the ankle.
a) Acute hemarthrosis 
b) Osteonecrosis 
c) Baker’s cyst 
d) pigmented villonodular synovitis 
e) rheumatoid arthritis


A Baker cyst is sometimes called a popliteal cyst.
When an excess of knee joint fluid is
compressed by the body weight between the
bones of the knee joint, it can become trapped
and separate from the joint to form the fluidfilled sac of a Baker cyst. The name of the cyst is
in memory of the physician who originally
described the condition, the British surgeon
William Morrant Baker (1839-1896).
Rheumatoid Hands




Mental retardation of varying severity is also
common.  Seizures in association with neuronal
migration disorders like pachygyria and
heterotopias are occasionally seen. Autism may
be seen in up to 10% of the patients.
Cardiac defects include ventricular septal
defects;  renal abnormalities comprise of
unilateral renal agenesis and horseshoe kidneys. 
Question 1






All of the following is correct about Atrial
Fibrillation except ?
A- The most common persistent arrhythmia.
B- Incidence or prevalence increases with age.
C- if hemodynamically unstable patient in ER
should be cardioverted .
D- Dabigatran( Pradaxa) new antithrombin
associated with Decresed GI bleed compared to
Warfarin





D- Dabigatran new antithrombin associated with
Decresed GI bleed compared to Warfarin
The correct answer is
Dabigatran new antithrombin associated with
INCREASED GI bleed compared to Warfarin
Atrial fibrillation is a common arrythmia
• In the FHS, the lifetime risk of atrial fibrillation (AFib) for adults age
40 is 26% for men and 23% for women1

• Prevalence of AFib in the US is estimated > 2.2 million adults with
an incidence > 70 000 2

• Prevalence increases steeply with age 2

1.Lloyd-Jones DM, et al. Circulation 2004;110:1042–
1046
AFib: a common but serious arrythmia
 AFib increases the risk of stroke 5-fold1
 The increase in risk of stroke is similar for

paroxysmal, persistent and permanent AFib2
 Strokes associated with AFib are usually more
severe than those from other causes, conferring
an increased risk of morbidity, mortality and
poor functional outcome1

1. Savelieva et al. Ann Med 2007;39:371–
2. Hart R et al. JACC 2000; 35:183
Dabigatran etexilate – a new, reversible, oral
DTI









Dabigatran etexilate is an oral prodrug, converted to
dabigatran, a potent reversible DTI
No interaction with food
No participation with CYP450
Predictable anticoagulant effect
Fixed dose
No need for routine coagulation monitoring
Very recently some genetic variation for response

28
Q4




A 68-year-old gentleman is admitted with an Anterior
myocardial infarction (MI) and receives thrombolysis,
aspirin, atenolol, atorvastatin and lisinopril. His ECG
shows good ST segment resolution.
The next day he develops some pain in the legs and a
dusky discolouration of the lower limbs. On closer
examination there is a diffuse petechial rash over the
lower limbs, particularly the feet, but all peripheral
pulses are palpable.



CBC showed elevated eosinophils
Q4








Which of the following is the most likely cause
for his current situation?
A)Polyarteritis Nodosa
B)Aspirin Allergy
C)Periphral Vascular Disease
D)Cholesterol Emboli
E)Post Thrombolysis Allergy
A4
D)Cholesterol Emboli
The  above patient is an arteriopath as suggested
by the acute Myocardial Infarction, and one day
after thrombolysis he develops a petechial rash
in the lower limbs with raised white cell count marked eosinophilia and raised IgE. This
suggests cholesterol embolisation
syndrome rather than allergy. 

Question 2


Catheter-delivered balloon expansion techniques are now the
treatment of choice for which one of the following lesions in
adults?
 A. Valvular pulmonic stenosis.
 B. Valvular aortic stenosis.
 C. Coarctation of the aorta.
 D. Ebstein's anomaly of the tricuspid valve.
 E. Severe mitral stenosis with Significant regurgitation.
Answer to Q 2


Comment
The correct answer is A.



Although catheter balloon valvuloplasty and aortoplasty have been attempted
in all these conditions, only pulmonary valvotomy has achieved a success level
consistent with being the treatment of choice in adults. Aortic stenosis
responds initially to balloon expansion and may serve as a bridge to valve
replacement surgery, but is associated with rapid restenosis. Success rates with
coarctation and Ebstein's anomaly are not uniform enough to displace surgery
except in selected patients.
Mitral stenosis in the absence of severe subvalvular disease can be
successfully treated by balloon valvuloplasty, but the presence of moderate to
severe regurgitation is an indication for surgery.
Q7


A 72 year old female with a history of breast cancer and tobacco use
complains of dizziness and dyspnea on exertion. Her heart sounds are distant
and her systolic blood pressure is noted to markedly decrease with inspiration.
Which of the following is the most likely diagnosis?



A) Constrictive cardiomyopathy
B) Mitral valve Stenosis
C) Congestive heart failure
D) Pulmonary embolus
E) Cardiac Tamponade





A7


E) Cardiac Tamponade
     Cancer is the most common cause of pericardial effusion and
when enough fluid accumulates in the pericardial space, cardiac
tamponade occurs. “Pulsus paradoxus” is when there is a
decrease in systolic blood pressure during inspiration due to
failure of the right ventricle to accept the normal increased
venous return that occurs with inspiration. This also results in a
“Kussmal’s sign” or elevated jugular venous distension during
inspiration (normally the opposite occurs). Treatment is with
emergent pericardiocentesis.
Q3


In which of the following diseases is pregnancy difficult, but not highly risky to mother and
fetus?
 A. Eisenmenger's syndrome.
 B. Primary pulmonary hypertension.
 C. Hypertrophic obstructive cardiomyopathy.
 D. Prior peripartum cardiomyopathy with heart failure.
 E. The Marfan syndrome with dilated aortic root.
Answer to Q3


 Comment
The correct answer is C.
The cardiovascular system must be able to handle a doubling of cardiac output during
pregnancy. Thus, cardiopulmonary diseases that obstruct blood flow are usually
contraindications to pregnancy because both the mother and fetus get inadequate blood
flow. Thus, obstruction to pulmonary flow due to the Eisenmenger reaction or primary
pulmonary hypertension fits into this category, but hypertrophic cardiomyopathy does not.
The increased cardiac output increases venous return to the left heart resulting in left
ventricular enlargement and less obstruction. In fact, during pregnancy the murmur of
hypertrophic obstructive cardiomyopathy may lessen or even disappear, causing the
diagnosis to be missed.
Prior peripartum cardiomyopathy with heart failure is a contraindication to pregnancy
because of the high incidence of recurrent failure and death.
Hormonal changes during pregnancy alter vascular walls, making them more distensible.
This is a normal mechanism to adapt to higher cardiac output; however, in the patient with
the Marfan syndrome and an enlarged aortic root, it can lead to increased wall stress and
aortic rupture or dissection.
Q8


A 50 year old gentleman is admitted to the CCU with an acute Inferior wall
infarction. Three hours after the admission his B.P is 90/50 mmHg. The
heart rate is 38 beats per minute with sinus rhythm. Which of the following
will be the most appropriate initial therapy?



A)Immediate insertion of temporary pacemaker
B)Intravenous administration of of Isoproterenol 5 micrograms/min
C)Intravenous administration of Dobutamine 0.35 mg/min
D) Administration of Intravenous Normal saline, 300 ml over 15 mins
E) Intravenous administration of atropine sulfate , 0.6 mg





A8


E) Intravenous administration of atropine
sulfate , 0.6 mg
Q9


A 50 year old gentleman with a history of hypertension and high cholesterol
presents to the emergency department with pain in chest for one hour. He
describes a substernal chest pressure associated with shortness of breath and
sweating. His ECG shows ST elevations consistent with myocardial
infarction. Which of the following laboratory results would be expected?



A) Elevated myoglobin, normal troponin I, and normal CK-MB
 B)Normal myoglobin, elevated troponin I, and normal CK-MB
 C)Normal myoglobin, normal troponin I, and elevated CK-MB
 D)Normal myoglobin, normal troponin I, and normal CK-MB




A9





A) Elevated myoglobin, normal troponin I, and normal
CK-MB
Myoglobin is a heme protein found in skeletal and
cardiac muscle that has attracted considerable interest
as an early marker of MI. Its low molecular weight
accounts for its early release profile:
myoglobin typically rises1- 2hours after onset of
infarction, peaks at 6-12 hours, and returns to normal
within 24-36 hours.
Q10


A 60 year old man with a histoty of Diabetes, hypertension and congestive
heart failure was brought to the ER after he complained of blurred vision and
headache. He was found to have Blood Pressure of 220/90 mm Hg. The
intern who examined her wanted to give her drug X, but the attending doctor
rejects this choice because of its tendency to cause compensatory tachycardia
and exacerbate fluid retention. The drug can also cause lupus like syndrome
with long term use. What is the mechanism of action of Drug X?
 A)Blocks Calcium Channels
 B)Increases Production of cGMP
 C)Decreases Production of cGMP
 D)Inhibits Angiotensin Converting Enzyme
 e)Inhibits carbonic Anhydrase
A 10







Hydralazine increases  (cGMP) levels, increasing the activity of
protein kinase G (PKG). Active PKG adds an inhibitory
phosphate to myosin light-chain kinase (MLCK) – a protein
involved in the activation of cross-bridge cycling (i.e.
contraction) in smooth muscle. This results in blood vessel
relaxation.
It dilates arterioles more than veins.
Hydralazine requires the endothelium to provide NO) thus only
provides the effects of NO in vivo with functional endothelium.
It will not work to vasodilate in vitro in an isolated blood vessel.
Activation of  has been suggested as a mechanism
Q 11


A 46 year old gentleman came to the Emergency
department with c/o sudden dyspnea. Patient
had similar episode of dyspnea two days earlier
which was associated with profuse sweating.
Patient is a reformed smoker and a social
drinker. ECG is displayed below. Let`s interpret
the ECG and decide the next step in the
management.
Sinus; PVCs; LAD; anterior symmetric T wave
inversion - and almost "Wellens"-like in V2,V3 - so
patient will probably need cath in short order
(looking for a tight LAD lesion ...).
Q 12


A 30 year old female with no significant past medical history is 42 weeks
pregnant and labor has just begun. She begins to complain of difficulty in
breathing which worsens throughout delivery to the point of requiring
intubation. She had never previously complained of any shortness of breath
or chest pain. Her temperature is 37.0, blood pressure 90/60, heart rate 135,
respirations 25, and oxygen saturation 100% on 60% FiO2 on the ventilator.
Physical examination reveals diffuse pulmonary rales, a II/IV early diastolic
murmur, and no lower extremity edema. Laboratory studies are normal. ECG
shows sinus tachycardia and left atrial enlargement. Her chest x-ray has
pulmonary edema. Which of the following is the most likely diagnosis?



A) Aortic Dissection
B)Dissection of Coronary Artery
C)Aortic valve Stenosis
D) Mitral Valve Stenosis




A12


D) Mitral Valve Stenosis
Q 13


A 60 year old gentleman with a history of dyslipidemia, hypertension and
CHF presents to the emergency department with acute onset swelling of the
lips, eyes, and tongue causing airway obstruction and respiratory distress. He
also has an urticarial rash. Which of the following medications is the likely
culprit of his current symptoms?



 A)Pindolol
 B)Clonidine
 C)Felodipine
 D)Captopril
 E)Methyldopa







 D)Captopril
Butter fly skin rash of
SLE
Q 14








A 72 year old female with no significant past medical history passes out while
exercising. She has intermittent exertional chest pains and dyspnea on
exertion as well. Her physical examination reveals a III/VI late-peaking
creshendo-decreshendo murmur at the right upper sternal border and a
III/VI holosystolic murmur at the apex. Her S2 heart sound is very soft and
her carotid upstroke is weak and delayed. Which of the following is most
likely causing her symptoms?
A) Aortic valve regurgitation
B) Aortic valve stenosis
C) Mitral valve regurgitation
D) Mitral valve stenosis
E) Mitral valve prolapse
A 14


Aortic valve stenosis (AS) presents with one of the classic triad: syncope (passing out),
exertional angina, or exertional dyspnea (from heart failure).  Over the age of 70 the
most likely cause is degenerative calcific aortic stenosis while under the age of 70 a
bicuspid aortic valve is the likely culprit. Rheumatic heart disease is the third leading
cause. Physical examination reveals a crescendo-decrescendo murmur at the aortic
listening post (right upper sternal border) which radiates to the carotids. The more
severe the aortic stenosis the later the peak of the murmur in systole and the softer the
A2 component of the S2 heart sound. The murmur is depicted below:



“Pulses parvus et tardus” is present upon carotid artery examination. Remember
parvus means weak and tardus means late. The murmur can radiate to the apex and
sound holosystolic mimicking mitral regurgitation (this is known as the Galiverdin
phenomenon). No medical treatment is available for aortic stenosis. Aortic valve
replacement surgically is indicated for anyone who is symptomatic.
Q 15








A 56 year old African American female with a history
of asthma is diagnosed with hypertension. Laboratory
studies reveal a creatinine of 3.0 mg/dL and a
potassium level of 5.1 mg/dL. Which of the following
medications is appropriate to treat her hypertension?
A) amlodipine
B) hydrochlorothiazide
C) enalapril
D) propranolol
E) spironolactone
Calcium channel blockers










Mechanism of Action
Calcium channel blockers (CCB) are a group of medications that act to inhibit either
central or peripheral calcium channels. This results in decreased calcium influx and
smooth muscle relaxation.
Dihydropyridine calcium channel blockers act predominantly in the periphery causing
arterial vasodilation.
Non-dihydropyridine calcium channel blockers act predominantly centrally to
decreased heart rate (chronotropy) and contractility (inotropy).
Indications
Calcium channel blockers are used to treat hypertension and stable angina.
The non-dihydropyridine calcium channel blockers are used to lower heart rate during
tachyarrhythmias especially atrial fibrillation and atrial flutter as well as during acute
coronary syndromes when beta-blockers are contraindicated.
Verapamil has been studied to treat hypertrophic obstructive cardiomyopathy as well
as multifocal atrial tachycardia.
Q 16








A 29 year old male with a history of ectopia lentis presents for a
routine physical examination. He is noted to have pectus
excavatum, pes planus, a high arched palate, and a positive wrist
and thumb sign. Which of the following cardiac disorders is
associated with his condition?
A) Aortic valve stenosis
B) Coarctation of the aorta
C) Mitral valve prolapse
D) Ventricular septal defect
E) Ebstein’s anomaly
A 16





C) Mitral valve prolapse
Marfan’s syndrome is associated with mitral valve prolapse and aortic
aneurysms. Specifically, the ascending aorta may dilated and predispose
patient’s to acute aortic dissection which can be fatal. Also, when the
ascending aorta dilates, the aortic valve annulus stretches causing the valve
leaflets to fail to coapt which results in aortic regurgitation.
Aortic stenosis (A) is not associated with Marfan’s syndrome and is caused be
either senile calcific degeneration of the valve or from a congenital bicuspid
aortic valve . Coartaction of the aorta (B) is associated with Turner’s
syndrome and presents with hypertension in the upper extremities and
hypotension in the lower extremities. “Rib notching” is seen on the chest xray.  Ventricular septal defects  (D) and Ebstein’s anomaly  (E) are not
associated with Marfan’s. 
Q4


In long-term follow-up of patients after surgical repair of
tetralogy of Fallot, the most common dysrhythmia observed is:
 A. Sinus bradycardia.
 B. Atrial fibrillation.
 C. Atrial tachycardia.
 D. Ventricular tachycardia.
 E. Junctional tachycardia.
Answer Q 4


Comment
The correct answer is D.



Complex ventricular arrhythmias often occur during long-term
follow-up of patients with tetralogy of Fallot. The incidence
correlates with age at repair and with higher residual
postoperative right ventricular systolic and end-diastolic
pressures. Sudden death accounts for a significant proportion of
the late mortality among these patients. In patients with
ventricular tachycardia, the site of origin is typically found to be
in the right ventricular outflow tract related to the previous
ventriculotomy and infundibular resection.
Question 5


A 42-year-old man is referred for evaluation of a systolic murmur. Your exam shows
normal carotid pulses, a prominent apical impulse, an early systolic sound, and a grade
III/VI mid-systolic murmur at the base. Respiration did not change the character of
these auscultatory findings. After an extrasystole, the systolic murmur increased in
intensity. Handgrip did not alter the systolic murmur. Valsalva decreased the intensity
of the murmur, and it returned to baseline intensity after seven heart beats.
 Which one of the following diagnoses is most likely?
 A. Congenital pulmonic stenosis.
 B. Innocent murmur.
 C. Mitral valve prolapse.
 D. Hypertrophic obstructive cardiomyopathy.
 E. Bicuspid aortic valve.
Answer to Q 5


Comment
The correct answer is E.



A systolic murmur that increases in intensity in the beat following an
extrasystole is usually due to turbulent flow out of the ventricles. Mitral
regurgitation is less likely because this murmur does not change following an
extrasystole. The murmur of hypertrophic obstructive myopathy usually
decreases with handgrip exercise. An innocent flow murmur is less likely
because of the presence of an early systolic sound and grade III intensity.
With pulmonic stenosis, there are characteristic changes in the intensity of the
murmur and the ejection sound during respiration. The ejection sound
establishes the diagnosis of an abnormal aortic valve, a bicuspid valve being
the most common abnormality.
Q6






Regarding the Renin-Angiotensin system, which if the
following is false ?
a. Chronic renal hypoperfusion leads to catecholamine release,
hypertension, cardiac hypertrophy and salt and water retention
b. Angiotensin Converting Enzyme (ACE) inhibitors work by
inhibiting Angiotensinogen cleavage to Angiotensin I
c. ACE-2 is not inhibited by Angiotensin Converting Enzyme
(ACE) inhibitors
d. Bradykinin is thought to mediate the cough associated with
ACE inhibitors
Answer to Q 6


b. Angiotensin Converting Enzyme (ACE)
inhibitors work by inhibiting Angiotensinogen
cleavage to Angiotensin I
Q7








Which of the following structures are not
found in the right atrium?
a. Tendon of Todaro
b. Moderator band
c. Koch's triangle
d. Pectinate muscle
c. Koch's triangle
d. Pectinate muscle
Answer to Q7





b. Moderator band
The moderator band, located in the right
ventricle, is a muscular bridge that connects
the distal septum and the right ventricular
free wall at the anterior papillary muscle.
The tendon of Todaro is a fibrous band
located between the valves of the inferior
vena cava and coronary sinus in the right
atrium.


Koch's triangle is located in the lower
medial portion of the right atrium, overlying
the AV node and the proximal His bundle.
Pectinate muscles arise from the crista
terminalis and course as bands on the right
atrial free wall.
32 years old male came with fever and acute CVA

Zoomed view at MV


M V vegetations ( endocarditis)
Major Criteria


1. Positive blood culture



Typical organism from 2 separate cultures- Viridans
streptococci, Strptococcus bovis, HACEK, S. aureus,
enterococci

OR


Persistently + blood culture- all of three/ majority 4
blood cultures

OR


Single +ve blood culture for Coxiella or phase IgG >
1:800


2. Evidence of endocardial involvement

ECHO- oscillating intracardiac mass
OR
 Abscess
OR
 New partial deheiscence of prosthetic valve/
new regurgitation

Minor criteria








Predisposing heart condition
Fever > 100.4F
Vascular- Emboli, pulmonary infarct, mycotic
aneurysm, Janeway lesions
Immune- Osler’s nodes, Roth spots,
glomerulonephritis
Microbiological evidence
Modified Duke Criteria


Definite IE









Microorganism (via culture or histology) in a valvular vegetation,
embolized vegetation, or intracardiac abscess
Histologic evidence of vegetation or intracardiac abscess
2 Major
1 Major + 3 minor
5 Minor

Possible IE



1 major and 1 minor
3 minor
Janeway Lesions

1. More specific
2. Erythematous, blanching macules
3. Nonpainful
4. Located on palms and soles
5. Microabscess of the dermis with marked necrosis and
inflammatory infiltrate not involving the epidermis.
Q8






2. Which of the following statements about the
pericardium is false?
a. The pericardium contains 80 to 90 mL of pericardial
fluid.
b. It is connected to the diaphragm, sternum, and
pleurae.
c. It encloses the entire main pulmonary artery.
d. The ligament of Marshall is a pericardial fold that
contains the remnant of the embryonic left superior
vena cava.
Answer Q 8




a. The pericardium contains 80 to 90 mL of
pericardial fluid.
The Correct answer is 10—50 mls
Q 18








A 45 year old female with a history of hypertension previously
controlled with diet and lifestyle modifications presents to here
primary care physician with increasing headaches. Her blood
pressure is 160/90. She is diagnosed with worsening migraine
headaches. Which of the following medications is appropriate to
treat her hypertension and headaches simultaneously?
A) hydrochlorothiazide
B) metoprolol
C) clonidine
D) methyldopa
E) lisinopril
A 18


Metoprolol is a lipid soluble beta-blocker which
can cross the blood-brain barrier easily and have
been shown to be effective in the prophylaxis of
migraine headaches as well as the treatment of
hypertension. They have slight sedating effects
and therefore can also be used for stage freight
or panic attacks as well.
Q9








5. Which of the following statements about
the tricuspid valve is true?
a. It has three leaflets: anterior, lateral, and
posterior.
b. It is the largest heart valve.
c. It is the most anterior heart valve.
d. The posterior leaflet is the largest and most
mobile
Dimeter of Valves
 AV:
 TRICUSPID

13 cm

 MITRAL

11 cm

 SEMILUNAR:
 PULMONIC 8 cm
 AORTIC

6 cm




Answer is b: The tricuspid valve is the largest
heart, whereas the pulmonic valve is the most
anterior.
The tricuspid valve consists of three leaflets,
anterior, septal, and posterior. The anterior
leaflet is the largest and most mobile. The
posterior leaflet is the smallest, whereas the
septal leaflet is the least mobile and occasionally
absent.
Q 10








1. Which of the following statements about the assessment
of the severity of a valvular abnormality are not true?
a. Severe acute aortic regurgitation results in a holodiastolic
murmur.
b. Severe aortic stenosis results in a long-duration and latepeaking systolic ejection murumur.
c. An S2-OS interval of less than 70 milliseconds is consistent
with severe mitral stenosis with normal heart rates and absence
of other left-sided valve disease.
d. Severe aortic stenosis may be present with a soft murmur.
e. The duration of a tricuspid stenosis murmur correlates with
the severity of stenosis
Answer to Q 10


The answer is a: The severity of AS is best determined
by the duration and time to peak of the murmur. Longduration and late peaking are consistent with severe AS.
The intensity of an AS murmur is dependent on the
stroke volume, so a soft murmur may occur with severe
AS and LV dysfunction. Similarly, the duration of TS or
MS murmurs are helpful in determining severity, as is
the duration of the S2-OS interval. However, the S2-OS
interval is also affected by heart rate and other valvular
lesions that affect LA and LV pressure. Severe
Q 19








A 67 year old male suffers a large myocardial infarction
complicated by sustained ventricular tachycardia. He is started on
a lidocaine infusion. He subsequently develops shock liver from
hypotension during the ventricular tachycardia and his serum
lidocaine levels become significantly elevated. Which of the
following is a manifestation of lidocaine toxicity?
A) Stroke
B) Seizures
C) Renal failure
D) Congestive heart failure
E) Hyperkalemia
A 19


B) Seizures



Lidocaine which is used to treat ventricular arrhythmias
such as ventricular tachycardia or ventricular
fibrillation, can easily reach toxic levels and can cause
seizures and may progress to coma and death.
Lidocaine at high levels first inhibits the inhibitory
neurons in the brain resulting in seizures. Eventually all
neurons are inhibited and coma ensues. No specific
treatment or antidote exists.


Q 21








A 62 year old female with a history of hypertension presents with
increasing shortness of breath, abdominal pain, and diarrhea.
Computed tomography reveals a mass in the appendix and
multiple nodules in the liver. Chest x-ray is normal. Serum 5hydroxyindoleacetic acid levels are elevated. Which of the
following cardiac disorders is she likely to have?
A) Aortic valve stenosis
B) Aortic valve regurgitation
C) Mitral valve stenosis
D) Mitral valve regurgitation
E) Tricuspid valve stenosis
A 21





E) Tricuspid valve stenosis
This patient has carcinoid syndrome which consists of diarrhea,
facial flushing, reactive airways causing shortness of breath, and
cardiac valvular disease specifically of right-sided heart valves
since the toxins produced by the tumor are filtered by the lungs
and never reach the left sided heart valves (unless pulmonary
metastasis are present).
Others are all left-sided heart valves which would not be affected
in carcinoid syndrome unless pulmonary metastasis are present
(which is rare).
Q 22







A 29 year old male with no past medical history has
been experiencing headaches for the past few months.
His blood pressure is noted to be 210/110. Physical
examination reveals an S4 heart sound and reduced
femoral pulses. Which of the following is associated
with his condition?
A) Atrial septal defect
B) Wolff-Parkinson-White syndrome
C) Bicuspid aortic valve
D) Mitral valve regurgitation
A 22





C) Bicuspid aortic valve
Coarctation of the aorta occurs when the congenital narrowing
of the aorta occurs. About two thirds of patients with coarctation
of the aorta have a bicuspid aortic valve as well. Depending on
the location of the narrowing differing presentations may occur.
Infantile coarctation of the aorta presents when the stenosis is
proximal or next to the ductus arteriosus. When the ductus
arteriosus closes (as it should in normal infants), a severe increase
in afterload occurs resulting in congestive heart failure (since
blood was normally able to traverse the patent ductus arteriosus
resulting in lower resistance, then suddenly is unable to).
Q 11









Which of the following will not cause a
continuous murmur?
a. An right coronary sinus to the right atrial
fistula.
b. A stenotic and regurgitant aortic valve.
c. A venous hum.
d. An aorta-to-pulmonary artery connection.
e. A high-grade coarctation of the aorta.
Answer to Q 1 1


The answer is b: A continuous murmur must
extend uninterrupted through S2. An aortapulmonary connection such as a PDA, a
coronary sinus-to-RA connection such as with a
ruptured sinus of Valsalva, a severe coarctation,
and a venous hum all result in continuous
murmurs. The combination of a systolic and
diastolic murmur of AS and AR is not
considered a continuous murmur.
Q 27








An 82 year old male with a history of long standing atrial
fibrillation and hypertension presents with increasing dyspnea on
exertion. Chest x-ray shows a honeycoming pattern and
pulmonary function testing shows a severe restrictive defect.
Which of the following is likely causing his symptoms?
A) Congestive heart failure
B) amiodarone
C) ramipril
D) sotalol
E) diltiazem
A 27








B) amiodarone
C) ramipril
D) sotalol
E) diltiazem
Amiodarone used to treat atrial fibrillation and ventricular arrhythmias, can
cause pulmonary fibrosis after long-term use.
Amiodarone also causes hypothyroidism, hyperthyroidism, and on rare
occasion liver failure.
Remember to check PFTs (pulmonary function tests), LFTs (liver function
tests), and TFTs (thyroid function tests) on all patients on amiodarone. Blue
man syndrome can occur as well due to deposition of amiodarone metabolites
in the skin resulting in a blue hue.
Q 12






Which of the following about differentiating AS
from HCM on examination is true?
a. The murmur of AS decreases and the murmur of
HCM increases with amyl nitrite.
b. Post-PVC, the pulse pressure of HCM decreases and
that of AS increases.
c. The murmur of AS and HCM decrease with
standing.
d. With Valsalva, the murmur of AS and HCM
decrease.
Answer to Q 12


The answer is b: Amyl nitrite results in an
increase in the murmur of AS and HOCM.
Valsalva (straining phase) and standing result in
an increase of the murmur of HOCM and
decreases the murmur of AS. Post-PVC, the
murmur of AS and HOCM increases but the
pulse pressure of HOCM decreases
(Brockenbrough phenomenon) while that of AS
increases.
Q 29


A 52 year old female is experiencing difficulty
hearing. Which of the following drugs may be
the cause?



A) bumetinide



B) clonidine
C) minoxidil
D) triamterene



A 29



A) bumetinide
Ototoxicity occurs with high dose loop diuretics such
as furosemide, bumetinide, or torsemide.
Aminoglycosides can cause similar hearing loss (also
nephrotoxicity). Remember that congenital ear
malformations are associated with congenital kidney
problems to help remember the connection between
medications that act on the kidney and cause hearing
loss such as loop diuretics.
Q 13







Regarding cardiac myxoma, all of the
following statements are true except:
a. It can arise anywhere within the heart.
b. It is the most common primary cardiac
tumor.
c. Approximately 75% occur in the left atrium.
d. Most are familial.
e. Myxomas may be associated with syncope,
TIA, or stroke
Q 14


Answer is d: Cardiac myxoma is the most
common primary cardiac tumor, accounting for
50% of primary cardiac tumors in adults.
Though it can be found anywhere in the heart,
the most common location is the left atrium
attached
Q 15








What of the following malignancies has the
highest likelihood of cardiac metastasis?
a. Lung cancer
b. Renal cell carcinoma
c. Melanoma
d. Breast cancer
e. Colon cancer
Answer to Q 15


Answer is c: The malignancy with the highest
likelihood of cardiac metastasis is melanoma,
though in absolute numbers, cardiac metastases
are more common, with lung and breast cancers
reflecting the higher incidence of these cancers.
Q 38







A 55 year old female with a history of congestive
heart failure is found to have breast cancer
requiring chemotherapy. Which of the following
chemotherapeutic agents should be avoided?
A) doxorubicin
B) bleomycin
C) paclitaxel
D) cyclophosphamide
A 38


A) doxorubicin



Doxorubicin and daunorubicin are anthracycline
chemotherapeutic agents that are well known to
cause systolic congestive heart failure especially at
higher doses and should be avoided if pre-existing heart
failure is present.
Bleomycin (B) can cause pulmonary fibrosis. Paclitaxel
(C) also causes pulmonary toxicity. Cyclophosphamide
(D) can cause hemorrhagic cystitis (resulting in
hematuria and bladder pain).  


Q 16







Digitalis is a cardiac glycoside that does all of the
following except:
a. indirectly activates a Na+/Ca++ exchanger found in
the cardiac cell membrane
b. inhibits the Na+/K+ ATPase found in the cardiac
cell membrane
c. increases chronotropy by increasing intracellular Ca+
+
d. leads to a sodium gradient across the cardiac cell
membrane that is favorable for Ca++ influx
e. increases parasympathetic tone
Answer to Q 16




Answer is c: Digitalis inhibits the Na+/K+ ATPase found in the
cardiac cell membrane which maintains a high intracellular K+
concentration and high extracellular Na+ concentration. Ca2+ is
removed from the cytosol into the extracellular fluid by a
sodium-calcium exchange pump driven by the pre-existing Na+
gradient. Inhibiting the Na+/K+ ATPase promotes enhanced
Na+/Ca2+ exchange ultimately leading to increased intracellular
Ca2+ available to the contractile apparatus increasing myocardial
contractility.
Chronotropy would not be increased with digitalis and in fact it
is often used for the opposite effect of heart rate control in
patients with atrial fibrillation.
Pic Quiz 45 years old with recent MI, This Skin lesion was
noted on his Leg
Answer 2 pic quiz


Necrobiosis lipoidica of DM



How is it treated ?
Q 17








All of the following are class III recommendations in the
treatment of unstable coronary syndromes, except:
a. Use of fibrinolytic therapy for non-ST-elevation acute
coronary syndromes
b. Use of abciximab for conservatively managed high-risk
patients who continue to have ischemic symptoms
c. The use of a low-molecular-weight heparin instead of
unfractionated heparin for conservatively managed unstable
coronary syndromes
d. Use of nitroglycerin within 24 hours of sildenafil (Viagra)
e. Invasive therapy in low-risk patients who present with a chest
pain syndrome
Answer to Q 17


Answer is c: There may be a marginal benefit of low-molecularweight heparin over unfractionated heparin for conservatively
managed patients, and this strategy is a class IIa
recommendation. Nitroglycerin should not be used within 24
hours from the last dose of sildenafil. Fibrinolytics should only
be used for ST-elevation myocardial infarctions. Ideally, high-risk
patients should be managed invasively, but for high-risk
individuals who defer invasive therapy or who have extensive comorbidities and continue to have ischemic symptoms, the use of
a glycoprotein IIb/IIIa inhibitor is a class IIa recommdation.
However, eptifibitide or tirofiban should be used in this setting,
while abciximab should be used only during invasive
management.
Q18








Which of the following are causes of secondary
angina OR Type 2 MI (Type 1 Sontaneous)?
a. An anemic patient from a gastrointestinal bleed
b. A dialysis patient with an arterio-venous fistula
c. A dyspneic patient with underlying emphysema
d. a and c
e. a, b, and c
Answer to Q 18


Answer is e: Anemia, anterior-venous shunting,
and hypoxemia can all cause demand ischemia.
Note that a left-arm arterio-venous fistula can
produce shunting as well as subclavian steal in
patients with a previous left internal mammary
artery graft.
Q 19








Which of the following is not included in the
differential diagnosis for
electrocardiographic ST elevations?
a. ST-elevation myocardial infarction
b. Left ventricular aneurysm
c. Hypokalemia
d. Pericarditis
e. Left ventricular hypertrophy
Answer to Q 19


Answer is c: Among the electrolyte
abnormalities, hyperkalemia, not hypokalemia
can cause ST elevations that mimic ST-elevation
myocardial infarctions.
Q 20








Risk factors for intracranial hemorrhage
during administration of fibrinolytics
include all of the following except:
a. Uncontrolled hypertension
b. Advanced age
c. Female gender
d. Preexisting coagulopathy
e. Morbid obesity
Answer to Q 20


Answer is e: Low body weight, not morbid
obesity, is a risk factor for intracranial
hemorrhage.
Q21








Digoxin toxicity may present with all the
following except?
a- Color visual defect
b-GI upset
c- Bidirectional VT
d-headache
e –all of the above
Answer to Q 21


e –all of the above
Q 22







Digoxin toxicity is more found except
a-young men
b- elederly females.
c-Renal impairement.
d-Liver impairment
e- a,d


e- a,d
Q23








All of the following clinical findings are
consistent with severe mitral stenosis except
A. atrial fibrillation
B. opening snap late after S2
C. pulmonary vascular congestion
D. pulsatile liver
E. right-ventricular heave
Answer to Q 23


B. opening snap late after S2
Q 24








A patient is found to have a holosystolic murmur on
physical examination.With deep inspiration, the
intensity of the murmur increases.This is consistent
with which of the following?
A. Atrial-septal defect
B. Austin Flint murmur
C. Carvallo’s sign
D. Chronic mitral regurgitation
E. Gallavardin effect
Answer to Q 24


C. Carvallo’s sign of TR



Carvallo's sign is a clinical sign found in
patients with tricuspid regurgitation.
The pansystolic murmur found in this condition
becomes louder during inspiration this sign
enables it to be distinguished from mitral
regurgitation





Pectus excavatum
Associated Marfan Syndrome
77 years old male with CHF,CRF
Q 25








Which of the following findings would be
suggestive of critical ischemia of the right foot?
A. Ankle-brachial index <0.3
B. Ankle-brachial index <0.9
C. Ankle-brachial index >1.2
D. Lack of palpable dorsalis pedis pulse
E. Presence of pitting edema of the extremities
Answer to Q 25


A. Ankle-brachial index <0.3
Q 26








All the following electrocardiogram (ECG)
findings re suggestive of left ventricular
hypertrophy except
A. (S in V1 + R in V5 or V6) >35 mm
B. R in aVL >11 mm
C. R in aVF >20 mm
D. (R in I + S in III) >25 mm
E. R in aVR >8 mm
Answer to Q 26


E. R in aVR >8 mm
Q 27








55 years old saudi male , investigated for recent
onset of exertional syncope , found to have AS ,
the most likely aetiology is ?
A. Age-related degeneration
B. Dyslipidemia
C. Glucose intolerance
D. Hypertension
E. Obesity
Answer to Q 27


A. Age-related degeneration
Q 28







In peripartum cardiomyopathy Which of the
following factors is the most valuable predictor
of mortality with subsequent pregnancies?
A. Age >30 years
B. African ancestry
C. Interpartum left ventricular function
D. Male child
Answer to Q 28


C. Interpartum left ventricular function
55 Years old lady with SOB & fatigability ECG & ECHO done
55 Years old lady with SOB & fatigability ECG & ECHO done

NSR , Low Voltage ECG
Moderate Size Pericardial effusion


Myxedema in Hypothyroidism



Atrial fibrillation might be the sole presentation
of thyrotoxicosis.
Atrial fibrillation may occur with
Hypothyroidism
Both Hypo-or Hyperthyroidism may cause
Cardiomyopathy, wait for 3- 6 months before
repeating ECHO after treament .



Q 29








HMG-CoA reductase inhibitor reduces coronary
events. This Medication will exert all the
following beneficial effects except?
A. direct action on atheroma progression
B. improvement in endothelial-dependent
vasomotion
C. long-term reduction of serum LDL
D. regression of existing coronary stenosis
E. stabilization of existing atherosclerotic lesions
Answer to Q 29


D. regression of existing coronary stenosis


tendon xanthomas at the wrists, knees, and
achilles, With ostial RCA


Haemorrhagic herpes zoster
Q 30






Which vein has the highest oxygen
saturation?
Which vein has the lowest oxygen
saturation?
In which situation is coronary sinus
saturation elevated?
Answer to Q 30


Renal vein has the highest oxygen saturation.
Hence the inferior vena caval (IVC) blood is
more saturated than superior vena caval blood.
The saturation is higher in the inferior vena cava
above the renal veins (high IVC) than the low
IVC.
Answer Q 30


Coronary sinus has the lowest oxygen saturation
as the oxygen extraction is maximum in the
coronary circulation. Coronary sinus saturation
does not fall with exercise as the extraction
reserve is fully utilized in the coronary
circulation and increase in oxygen consumption
can occur only by increasing the flow
Q 31



Janeway lesion:
a) Tender
b) Nodular
c) Most common in trunk
d) Blanches on pressure
A 31



Answer: d) Blanches on pressure
Nodular and tender lesions are Osler’s nodes.
Janeway lesions are seen on the palms and soles.
They are non tender, macular lesions. These are
lesions seen in infective endocarditis
Q 32




HACEK organisms causing endocarditis
includes:
a) Cardiobacterium
b) Acinetobacter
c) E. coli
d) Klebsiella
A 32



Answer: a) Cardiobacterium
HACEK is an acronym for a group of gram-negative
bacilli: Haemophilus species (Haemophilus
parainfluenzae, Haemophilus aphrophilus,
Haemophilus paraphrophilus), Actinobacillus
actinomycetemcomitans, Cardiobacterium hominis,
Eikenella corrodens, and Kingella species. They have a
higher potential to produce endocarditis and is
responsible for about three percent of native valve
endocarditis
Q 33






Which of the following statements about
infective endocarditis is wrong?
a) Splenomegaly is more common in acute than
sub acute infective endocarditis
b) Disease manifestation occurs within two
weeks of the cause of bacteremia
c) Cerebral embolism is more common in the
middle cerebral artery territory
d) 5% chance of intracranial hemorrhage
A 33


a) Splenomegaly is more common in acute than
sub acute infective endocarditis
Q34



Gene defective in Marfan syndrome:
a) Collagen
b) Fibrillin
c) Elastin
d) Cathepsin
A 34



b) Fibrillin
Marfan syndrome is caused by defects in a gene
called fibrillin-1 (FBN1), located on
chromosome 15.
Q 35




For blood pressure measurement, least
important is:
a) Arm cuff at heart level
b) Manometer at heart level
c) Arm cuff >80% arm circumference
d) Manometer should be vertical
A 35


b) Manometer at heart level
Manometer need not be at heart level as the difference involved
will be only the weight of the air column in the tubing, which is
negligible compared to that of mercury. But the mercury level
should be at the level of your eye to avoid parallax error. A
slanting mercury column will increase the reading. Position of
arm cuff is important as level difference will alter the recorded
pressure which will depend on the column of blood in the vessel
above or below the heart level. Cuffs which are shorter will not
provide adequate compression during inflation and is likely to
record falsely elevated pressures
Q 36



QT interval shortens in:
a) Hypothermia
b) Hypokalemia
c) Hypocalcemia
d) Acidosis
A36



Answer: d) Acidosis
Acidosis is associated with shift of potassium
out of the cells and hyperkalemia. QT
prolongation in hypocalcemia is due to ST
segment prolongation. Digitalis and congenital
short QT syndrome are other causes of a short
QT interval.
60 years old male with CHF
Q 37




According to BMI (body mass index), the cut
off for obesity is:
a) 25 Kg/m2
b) 30 Kg/m2
c) 35 Kg/m2
d) 40 Kg/m2
A37








Answer: b) 30 Kg/m2
The International Classification of adult underweight, overweight
and obesity according to BMI (World Health Organization) is as
follows:
Underweight <18.50
Normal range 18.50 – 24.99
Overweight ≥25.00
Pre-obese 25.00 – 29.99
Obese ≥30.00
Obese class I: 30.00 – 34.99
Obese class II: 35.00 – 39.99
Obese class III: ≥40.00
Q 38
A 38

Tamponade can be acute or subacute, depending on the etiology:
Acute: Rapid accumulation (usually blood) within a stiff, noncompliant pericardium
Subacute: Gradual increase of a preexisting effusion, with limited
accommodative pericardial stretch
Q 39
A39
Q 40
A 40
Q 41
A 41


D- VMA
Q 42 ???????
Q 42
A 42


Dextrocardia
Associated with situs inversus and kartageners
syndrome.



Sometimes polysplenism


Osteogenesis imperfecta
Blue sclera
Q 43 ?????????? 25 years old with Flu
A 43








Diffuse ST elevation with PR segment
depression Inferior Leads.This is Acute
Pericarditis.
Treatment First Line
A- High dose ASA 2- grams per day
B- Paracetamol
C-Cholchicine
Avoid NSAIDs and steroids at least as first line ,
in post MI.
Q 44 ?????? 60 male with CAD
Where is the delta waves
WPW syndrome with pseudo
inferior MI pattern
A 44








WPW
Next step is Echo is needed because of
increased incidence of associated structural heart
disease with WPW.
Treatment
A- Asymptomatic Leave Alone
B- Symptomatic offer EPS + ablation
C- found and difficult to ablate in Ebstein
anomaly
Q 45


Atrial Flutter with 2:1 Block.
B




25 year old female has a butterfly rash on her
face, photosensitivity and oral ulcers. She is
diagnosed with SLE. Which of the following is
another characteristic of SLE?
a) polyuria 
b) polydipsia 
c) polyphagia 
d) pericarditis 
e) Philadelphia chromosom


d) pericarditis 




You are reviewing a patient in the intensive care
unit with acute renal failure. Which antimicrobial
agent is LEAST likely to require a dose
adjustment in renal failure?
a) gentamicin
b) amitriptyline
c) carbonic anhydrase inhibitors
d) streptomycin
e) erythromycin


e) erythromycin




A 44 year old woman with a long history of
multiple sclerosis complains of severe pain on
the right side of her face. The most likely
pathology is in the
a) the right facial nerve
b) the right long thoracic nerve
c) the right trigeminal nerve
d) the left trigeminal nerve
e) the right recurrent laryngeal nerve
Correct Answer: C
Pain typical of trigeminal neuralgia occasionally affects
patients with lesions in the brain stem as a result of
multiple sclerosis. They may also occur with vasculitis
involving the descending root of the fifth cranial nerve.
Trigeminal neuralgia usually occurs after other symptoms
of MS. Of all patients with MS, however, about 10%
have facial pain as a presentation, and other symptoms of
MS may not appear for 6 yea

Thank

much

you very

Mais conteúdo relacionado

Mais procurados

PCI vs. thrombolysis
PCI vs. thrombolysisPCI vs. thrombolysis
PCI vs. thrombolysisAmeel Yaqo
 
Invasive evaluation timing in nstemi (1)
Invasive evaluation  timing in nstemi (1)Invasive evaluation  timing in nstemi (1)
Invasive evaluation timing in nstemi (1)Supratip Kundu
 
Carotid revascularization in cad patients
Carotid revascularization in cad patientsCarotid revascularization in cad patients
Carotid revascularization in cad patientsDIPAK PATADE
 
Ventricular septal rupture with cardiogenic shock follows by Inferior AMI
Ventricular septal rupture with cardiogenic shock follows by Inferior AMIVentricular septal rupture with cardiogenic shock follows by Inferior AMI
Ventricular septal rupture with cardiogenic shock follows by Inferior AMIHan Naung Tun
 
Remote Ischemic Conditioning - Dr. Robert Kloner
Remote Ischemic Conditioning - Dr. Robert KlonerRemote Ischemic Conditioning - Dr. Robert Kloner
Remote Ischemic Conditioning - Dr. Robert KlonerEndothelix
 
Thrombolytics for Pulmonary Embolism
Thrombolytics for Pulmonary EmbolismThrombolytics for Pulmonary Embolism
Thrombolytics for Pulmonary Embolismchrispartyka
 
Revascularization in heart faliure seminar
Revascularization in heart faliure seminarRevascularization in heart faliure seminar
Revascularization in heart faliure seminarAnkit Jain
 
Role of DVT surveillance in TBI/SCI
Role of DVT surveillance in TBI/SCIRole of DVT surveillance in TBI/SCI
Role of DVT surveillance in TBI/SCIAmit Agrawal
 
arrhythmogenic right ventricular dysplasia/Cardiomyopathy
arrhythmogenic right ventricular dysplasia/Cardiomyopathyarrhythmogenic right ventricular dysplasia/Cardiomyopathy
arrhythmogenic right ventricular dysplasia/CardiomyopathyAnthony Kaviratne
 
Coronary revascularization in diabetes mellitus and multivessel cad
Coronary revascularization in diabetes mellitus and multivessel cadCoronary revascularization in diabetes mellitus and multivessel cad
Coronary revascularization in diabetes mellitus and multivessel cadSatyam Rajvanshi
 
Revascularisation strategies
Revascularisation strategiesRevascularisation strategies
Revascularisation strategiesdrucsamal
 

Mais procurados (20)

Presentation1
Presentation1Presentation1
Presentation1
 
PCI vs. thrombolysis
PCI vs. thrombolysisPCI vs. thrombolysis
PCI vs. thrombolysis
 
Abeer elnakera
Abeer elnakeraAbeer elnakera
Abeer elnakera
 
Invasive evaluation timing in nstemi (1)
Invasive evaluation  timing in nstemi (1)Invasive evaluation  timing in nstemi (1)
Invasive evaluation timing in nstemi (1)
 
Carotid revascularization in cad patients
Carotid revascularization in cad patientsCarotid revascularization in cad patients
Carotid revascularization in cad patients
 
Ventricular septal rupture with cardiogenic shock follows by Inferior AMI
Ventricular septal rupture with cardiogenic shock follows by Inferior AMIVentricular septal rupture with cardiogenic shock follows by Inferior AMI
Ventricular septal rupture with cardiogenic shock follows by Inferior AMI
 
Remote Ischemic Conditioning - Dr. Robert Kloner
Remote Ischemic Conditioning - Dr. Robert KlonerRemote Ischemic Conditioning - Dr. Robert Kloner
Remote Ischemic Conditioning - Dr. Robert Kloner
 
Thrombolytics for Pulmonary Embolism
Thrombolytics for Pulmonary EmbolismThrombolytics for Pulmonary Embolism
Thrombolytics for Pulmonary Embolism
 
ARVD
ARVDARVD
ARVD
 
Arrhythmogenic right ventricular 2003
Arrhythmogenic right ventricular 2003Arrhythmogenic right ventricular 2003
Arrhythmogenic right ventricular 2003
 
Practice Intersection: How I Approach Thrombus in My Daily Clinical Practice
Practice Intersection: How I Approach Thrombus in My Daily Clinical Practice Practice Intersection: How I Approach Thrombus in My Daily Clinical Practice
Practice Intersection: How I Approach Thrombus in My Daily Clinical Practice
 
Vte 2014
Vte 2014Vte 2014
Vte 2014
 
Revascularization in heart faliure seminar
Revascularization in heart faliure seminarRevascularization in heart faliure seminar
Revascularization in heart faliure seminar
 
Final thrombus burden
Final thrombus burdenFinal thrombus burden
Final thrombus burden
 
Pericarditis
PericarditisPericarditis
Pericarditis
 
Role of DVT surveillance in TBI/SCI
Role of DVT surveillance in TBI/SCIRole of DVT surveillance in TBI/SCI
Role of DVT surveillance in TBI/SCI
 
arrhythmogenic right ventricular dysplasia/Cardiomyopathy
arrhythmogenic right ventricular dysplasia/Cardiomyopathyarrhythmogenic right ventricular dysplasia/Cardiomyopathy
arrhythmogenic right ventricular dysplasia/Cardiomyopathy
 
Coronary revascularization in diabetes mellitus and multivessel cad
Coronary revascularization in diabetes mellitus and multivessel cadCoronary revascularization in diabetes mellitus and multivessel cad
Coronary revascularization in diabetes mellitus and multivessel cad
 
Revascularisation strategies
Revascularisation strategiesRevascularisation strategies
Revascularisation strategies
 
卵円孔開存
卵円孔開存卵円孔開存
卵円孔開存
 

Destaque

Ecg Part 1
Ecg Part 1Ecg Part 1
Ecg Part 1hospital
 
Important MCQs of CPT Accounts ICAI Module 2009
Important MCQs of  CPT Accounts ICAI Module  2009Important MCQs of  CPT Accounts ICAI Module  2009
Important MCQs of CPT Accounts ICAI Module 2009VXplain
 
Ecg part introduction
Ecg part introductionEcg part introduction
Ecg part introductionhospital
 
Cardiology board mc qs.ppt1
Cardiology board mc qs.ppt1Cardiology board mc qs.ppt1
Cardiology board mc qs.ppt1hospital
 
Ecg made easy
Ecg made easyEcg made easy
Ecg made easyALAA AWN
 
Cardiology cases ppt 4slideshare
Cardiology cases ppt 4slideshareCardiology cases ppt 4slideshare
Cardiology cases ppt 4slidesharehospital
 
ECG: Indication and Interpretation
ECG: Indication and InterpretationECG: Indication and Interpretation
ECG: Indication and InterpretationRakesh Verma
 
Endocrine emergencies MCQ's/SBA
Endocrine emergencies MCQ's/SBAEndocrine emergencies MCQ's/SBA
Endocrine emergencies MCQ's/SBASteve Mathieu
 
Ecg test 2
Ecg test 2Ecg test 2
Ecg test 2hospital
 
Jugular venous pressure
Jugular venous pressureJugular venous pressure
Jugular venous pressureBasem Enany
 
Mcq in cardiology 2015 magdi sasi
Mcq  in cardiology  2015  magdi  sasiMcq  in cardiology  2015  magdi  sasi
Mcq in cardiology 2015 magdi sasicardilogy
 
Rapid Review Of Embryology, First Edition, Ahmed M. Ayesh, 2017
Rapid Review Of Embryology, First Edition, Ahmed M. Ayesh, 2017Rapid Review Of Embryology, First Edition, Ahmed M. Ayesh, 2017
Rapid Review Of Embryology, First Edition, Ahmed M. Ayesh, 2017Ahmed Ayesh
 
Collections of CS Multiple Choice Questions
Collections of CS Multiple Choice QuestionsCollections of CS Multiple Choice Questions
Collections of CS Multiple Choice QuestionsShusil Baral
 
DBMS Multiple Choice Questions
DBMS Multiple Choice QuestionsDBMS Multiple Choice Questions
DBMS Multiple Choice QuestionsShusil Baral
 
How to create multiple choice questions
How to create multiple choice questionsHow to create multiple choice questions
How to create multiple choice questionsJennifer Morrow
 

Destaque (19)

ECG reading
ECG readingECG reading
ECG reading
 
Ecg Part 1
Ecg Part 1Ecg Part 1
Ecg Part 1
 
Important MCQs of CPT Accounts ICAI Module 2009
Important MCQs of  CPT Accounts ICAI Module  2009Important MCQs of  CPT Accounts ICAI Module  2009
Important MCQs of CPT Accounts ICAI Module 2009
 
Ecg part introduction
Ecg part introductionEcg part introduction
Ecg part introduction
 
Cardiology board mc qs.ppt1
Cardiology board mc qs.ppt1Cardiology board mc qs.ppt1
Cardiology board mc qs.ppt1
 
Heart failure ppt
Heart failure ppt Heart failure ppt
Heart failure ppt
 
Ecg made easy
Ecg made easyEcg made easy
Ecg made easy
 
Cardiology cases ppt 4slideshare
Cardiology cases ppt 4slideshareCardiology cases ppt 4slideshare
Cardiology cases ppt 4slideshare
 
ECG: Indication and Interpretation
ECG: Indication and InterpretationECG: Indication and Interpretation
ECG: Indication and Interpretation
 
Endocrine emergencies MCQ's/SBA
Endocrine emergencies MCQ's/SBAEndocrine emergencies MCQ's/SBA
Endocrine emergencies MCQ's/SBA
 
Ecg test 2
Ecg test 2Ecg test 2
Ecg test 2
 
Jugular venous pressure
Jugular venous pressureJugular venous pressure
Jugular venous pressure
 
Mcq in cardiology 2015 magdi sasi
Mcq  in cardiology  2015  magdi  sasiMcq  in cardiology  2015  magdi  sasi
Mcq in cardiology 2015 magdi sasi
 
Multiple choice-questions
Multiple choice-questionsMultiple choice-questions
Multiple choice-questions
 
shock
shockshock
shock
 
Rapid Review Of Embryology, First Edition, Ahmed M. Ayesh, 2017
Rapid Review Of Embryology, First Edition, Ahmed M. Ayesh, 2017Rapid Review Of Embryology, First Edition, Ahmed M. Ayesh, 2017
Rapid Review Of Embryology, First Edition, Ahmed M. Ayesh, 2017
 
Collections of CS Multiple Choice Questions
Collections of CS Multiple Choice QuestionsCollections of CS Multiple Choice Questions
Collections of CS Multiple Choice Questions
 
DBMS Multiple Choice Questions
DBMS Multiple Choice QuestionsDBMS Multiple Choice Questions
DBMS Multiple Choice Questions
 
How to create multiple choice questions
How to create multiple choice questionsHow to create multiple choice questions
How to create multiple choice questions
 

Semelhante a Cardiology Lecture to IM Board Exam oriented MCQS Picture Quiz Explanatory notes

CARDIAC PATHOLOGY (MCQ QUESTIONS & ANSWERS)
CARDIAC PATHOLOGY (MCQ QUESTIONS & ANSWERS)CARDIAC PATHOLOGY (MCQ QUESTIONS & ANSWERS)
CARDIAC PATHOLOGY (MCQ QUESTIONS & ANSWERS)Suraj Dhara
 
Congenital heart disease
Congenital heart diseaseCongenital heart disease
Congenital heart diseasesurendra sharma
 
Images of Vascular Medicine Symposia - The CRUDEM Foundation
Images of Vascular Medicine Symposia - The CRUDEM FoundationImages of Vascular Medicine Symposia - The CRUDEM Foundation
Images of Vascular Medicine Symposia - The CRUDEM FoundationThe CRUDEM Foundation
 
Clinical Cases In Cardiology
Clinical Cases In CardiologyClinical Cases In Cardiology
Clinical Cases In Cardiologyhospital
 
Approach to Chest pain
Approach to Chest pain Approach to Chest pain
Approach to Chest pain ahm732
 
Approach to heart failure cases
Approach to heart failure casesApproach to heart failure cases
Approach to heart failure caseshospital
 
Nuclear cases Saturday 2023.pptx
Nuclear cases Saturday 2023.pptxNuclear cases Saturday 2023.pptx
Nuclear cases Saturday 2023.pptxhospital
 
atrial fibrillation talk 2009
atrial fibrillation talk 2009atrial fibrillation talk 2009
atrial fibrillation talk 2009johnhakim
 
Acute coronary syndrome
Acute coronary syndromeAcute coronary syndrome
Acute coronary syndromeLih Yin Chong
 
IVMS-Cardiovascular Disorders Examination with Answers and Rationale
IVMS-Cardiovascular Disorders Examination with Answers and RationaleIVMS-Cardiovascular Disorders Examination with Answers and Rationale
IVMS-Cardiovascular Disorders Examination with Answers and RationaleImhotep Virtual Medical School
 
Management of congenital heart disease in infants
Management of congenital heart disease in infantsManagement of congenital heart disease in infants
Management of congenital heart disease in infantsSMSRAZA
 
Cardiac Amyloidosis Final Presentation.pdf
Cardiac Amyloidosis Final Presentation.pdfCardiac Amyloidosis Final Presentation.pdf
Cardiac Amyloidosis Final Presentation.pdfJonathanStrandberg1
 
Cardiomyopathies & Valvular Disorders - BMH/Tele
Cardiomyopathies & Valvular Disorders - BMH/TeleCardiomyopathies & Valvular Disorders - BMH/Tele
Cardiomyopathies & Valvular Disorders - BMH/TeleTeleClinEd
 

Semelhante a Cardiology Lecture to IM Board Exam oriented MCQS Picture Quiz Explanatory notes (20)

CARDIAC PATHOLOGY (MCQ QUESTIONS & ANSWERS)
CARDIAC PATHOLOGY (MCQ QUESTIONS & ANSWERS)CARDIAC PATHOLOGY (MCQ QUESTIONS & ANSWERS)
CARDIAC PATHOLOGY (MCQ QUESTIONS & ANSWERS)
 
Congenital heart disease
Congenital heart diseaseCongenital heart disease
Congenital heart disease
 
Congenital heart disease
Congenital heart diseaseCongenital heart disease
Congenital heart disease
 
Images of Vascular Medicine Symposia - The CRUDEM Foundation
Images of Vascular Medicine Symposia - The CRUDEM FoundationImages of Vascular Medicine Symposia - The CRUDEM Foundation
Images of Vascular Medicine Symposia - The CRUDEM Foundation
 
Pediatrics
Pediatrics Pediatrics
Pediatrics
 
Cardio1
Cardio1Cardio1
Cardio1
 
Clinical Cases In Cardiology
Clinical Cases In CardiologyClinical Cases In Cardiology
Clinical Cases In Cardiology
 
Approach to Chest pain
Approach to Chest pain Approach to Chest pain
Approach to Chest pain
 
ECG in young
ECG in youngECG in young
ECG in young
 
Uremic Pericarditis
Uremic PericarditisUremic Pericarditis
Uremic Pericarditis
 
Congenital heart-disease2787
Congenital heart-disease2787Congenital heart-disease2787
Congenital heart-disease2787
 
Approach to heart failure cases
Approach to heart failure casesApproach to heart failure cases
Approach to heart failure cases
 
Acute MI - NSTEMI
Acute MI - NSTEMIAcute MI - NSTEMI
Acute MI - NSTEMI
 
Nuclear cases Saturday 2023.pptx
Nuclear cases Saturday 2023.pptxNuclear cases Saturday 2023.pptx
Nuclear cases Saturday 2023.pptx
 
atrial fibrillation talk 2009
atrial fibrillation talk 2009atrial fibrillation talk 2009
atrial fibrillation talk 2009
 
Acute coronary syndrome
Acute coronary syndromeAcute coronary syndrome
Acute coronary syndrome
 
IVMS-Cardiovascular Disorders Examination with Answers and Rationale
IVMS-Cardiovascular Disorders Examination with Answers and RationaleIVMS-Cardiovascular Disorders Examination with Answers and Rationale
IVMS-Cardiovascular Disorders Examination with Answers and Rationale
 
Management of congenital heart disease in infants
Management of congenital heart disease in infantsManagement of congenital heart disease in infants
Management of congenital heart disease in infants
 
Cardiac Amyloidosis Final Presentation.pdf
Cardiac Amyloidosis Final Presentation.pdfCardiac Amyloidosis Final Presentation.pdf
Cardiac Amyloidosis Final Presentation.pdf
 
Cardiomyopathies & Valvular Disorders - BMH/Tele
Cardiomyopathies & Valvular Disorders - BMH/TeleCardiomyopathies & Valvular Disorders - BMH/Tele
Cardiomyopathies & Valvular Disorders - BMH/Tele
 

Mais de hospital

2023 HF p EF.pptx
2023 HF p EF.pptx2023 HF p EF.pptx
2023 HF p EF.pptxhospital
 
SURGICAL MANGEMNT of VHD 2023fffffffff.pptx
SURGICAL MANGEMNT of VHD 2023fffffffff.pptxSURGICAL MANGEMNT of VHD 2023fffffffff.pptx
SURGICAL MANGEMNT of VHD 2023fffffffff.pptxhospital
 
Reading and interpreting Holter2023.pptx
Reading and interpreting Holter2023.pptxReading and interpreting Holter2023.pptx
Reading and interpreting Holter2023.pptxhospital
 
Complications_of_Diabetes.ppt
Complications_of_Diabetes.pptComplications_of_Diabetes.ppt
Complications_of_Diabetes.ppthospital
 
SURGICAL MANGEMNT of VHD 2023.pptx
SURGICAL MANGEMNT of VHD 2023.pptxSURGICAL MANGEMNT of VHD 2023.pptx
SURGICAL MANGEMNT of VHD 2023.pptxhospital
 
the worst Medical Errors and how to mange them.pptx
the worst  Medical Errors and how to mange them.pptxthe worst  Medical Errors and how to mange them.pptx
the worst Medical Errors and how to mange them.pptxhospital
 
Radionuclide Imaging of Cardiac Amyloidosis and SarcoidosisFELLOWS LECTUREupd...
Radionuclide Imaging of Cardiac Amyloidosis and SarcoidosisFELLOWS LECTUREupd...Radionuclide Imaging of Cardiac Amyloidosis and SarcoidosisFELLOWS LECTUREupd...
Radionuclide Imaging of Cardiac Amyloidosis and SarcoidosisFELLOWS LECTUREupd...hospital
 
End Organ Damage In HypertensionDARB.pptx
End Organ Damage In HypertensionDARB.pptxEnd Organ Damage In HypertensionDARB.pptx
End Organ Damage In HypertensionDARB.pptxhospital
 
DELIVER delivered 2022.pptx
DELIVER delivered 2022.pptxDELIVER delivered 2022.pptx
DELIVER delivered 2022.pptxhospital
 
Percutaneous Revascularization for Ischemic Left Ventricular Dysfunction REV...
Percutaneous Revascularization for Ischemic Left Ventricular Dysfunction  REV...Percutaneous Revascularization for Ischemic Left Ventricular Dysfunction  REV...
Percutaneous Revascularization for Ischemic Left Ventricular Dysfunction REV...hospital
 
Acetazolamide in Acute Decompensated Heart Failure with Volume ADVOR.pptx
Acetazolamide in Acute Decompensated Heart Failure with Volume ADVOR.pptxAcetazolamide in Acute Decompensated Heart Failure with Volume ADVOR.pptx
Acetazolamide in Acute Decompensated Heart Failure with Volume ADVOR.pptxhospital
 
MRCP SUDAN QUESTIONS.pptx
MRCP SUDAN QUESTIONS.pptxMRCP SUDAN QUESTIONS.pptx
MRCP SUDAN QUESTIONS.pptxhospital
 
DM LECTURE PROJECT.pptx
DM LECTURE  PROJECT.pptxDM LECTURE  PROJECT.pptx
DM LECTURE PROJECT.pptxhospital
 
Learning case of strongly positive PET-CT Rubidium 82 on all parameters.ppt
Learning case of strongly positive PET-CT Rubidium 82 on all parameters.pptLearning case of strongly positive PET-CT Rubidium 82 on all parameters.ppt
Learning case of strongly positive PET-CT Rubidium 82 on all parameters.ppthospital
 
The Role of PET-CT imaging in the diagnosis of cardiac sarcoidosis post cardi...
The Role of PET-CT imaging in the diagnosis of cardiac sarcoidosis post cardi...The Role of PET-CT imaging in the diagnosis of cardiac sarcoidosis post cardi...
The Role of PET-CT imaging in the diagnosis of cardiac sarcoidosis post cardi...hospital
 
Diabetes mellitus and vascular disease 2022 FINALD.pptx
Diabetes mellitus and vascular disease 2022 FINALD.pptxDiabetes mellitus and vascular disease 2022 FINALD.pptx
Diabetes mellitus and vascular disease 2022 FINALD.pptxhospital
 
Nuc part BBBB.pptx
Nuc part BBBB.pptxNuc part BBBB.pptx
Nuc part BBBB.pptxhospital
 
Non Invasive testing of myocardial ischemia AA.pptx
Non Invasive testing of myocardial ischemia AA.pptxNon Invasive testing of myocardial ischemia AA.pptx
Non Invasive testing of myocardial ischemia AA.pptxhospital
 
Inherited aortopathy2022
Inherited aortopathy2022Inherited aortopathy2022
Inherited aortopathy2022hospital
 
The role of cardiac devise therapy in heart2021 ihab
The role of cardiac devise therapy in heart2021 ihabThe role of cardiac devise therapy in heart2021 ihab
The role of cardiac devise therapy in heart2021 ihabhospital
 

Mais de hospital (20)

2023 HF p EF.pptx
2023 HF p EF.pptx2023 HF p EF.pptx
2023 HF p EF.pptx
 
SURGICAL MANGEMNT of VHD 2023fffffffff.pptx
SURGICAL MANGEMNT of VHD 2023fffffffff.pptxSURGICAL MANGEMNT of VHD 2023fffffffff.pptx
SURGICAL MANGEMNT of VHD 2023fffffffff.pptx
 
Reading and interpreting Holter2023.pptx
Reading and interpreting Holter2023.pptxReading and interpreting Holter2023.pptx
Reading and interpreting Holter2023.pptx
 
Complications_of_Diabetes.ppt
Complications_of_Diabetes.pptComplications_of_Diabetes.ppt
Complications_of_Diabetes.ppt
 
SURGICAL MANGEMNT of VHD 2023.pptx
SURGICAL MANGEMNT of VHD 2023.pptxSURGICAL MANGEMNT of VHD 2023.pptx
SURGICAL MANGEMNT of VHD 2023.pptx
 
the worst Medical Errors and how to mange them.pptx
the worst  Medical Errors and how to mange them.pptxthe worst  Medical Errors and how to mange them.pptx
the worst Medical Errors and how to mange them.pptx
 
Radionuclide Imaging of Cardiac Amyloidosis and SarcoidosisFELLOWS LECTUREupd...
Radionuclide Imaging of Cardiac Amyloidosis and SarcoidosisFELLOWS LECTUREupd...Radionuclide Imaging of Cardiac Amyloidosis and SarcoidosisFELLOWS LECTUREupd...
Radionuclide Imaging of Cardiac Amyloidosis and SarcoidosisFELLOWS LECTUREupd...
 
End Organ Damage In HypertensionDARB.pptx
End Organ Damage In HypertensionDARB.pptxEnd Organ Damage In HypertensionDARB.pptx
End Organ Damage In HypertensionDARB.pptx
 
DELIVER delivered 2022.pptx
DELIVER delivered 2022.pptxDELIVER delivered 2022.pptx
DELIVER delivered 2022.pptx
 
Percutaneous Revascularization for Ischemic Left Ventricular Dysfunction REV...
Percutaneous Revascularization for Ischemic Left Ventricular Dysfunction  REV...Percutaneous Revascularization for Ischemic Left Ventricular Dysfunction  REV...
Percutaneous Revascularization for Ischemic Left Ventricular Dysfunction REV...
 
Acetazolamide in Acute Decompensated Heart Failure with Volume ADVOR.pptx
Acetazolamide in Acute Decompensated Heart Failure with Volume ADVOR.pptxAcetazolamide in Acute Decompensated Heart Failure with Volume ADVOR.pptx
Acetazolamide in Acute Decompensated Heart Failure with Volume ADVOR.pptx
 
MRCP SUDAN QUESTIONS.pptx
MRCP SUDAN QUESTIONS.pptxMRCP SUDAN QUESTIONS.pptx
MRCP SUDAN QUESTIONS.pptx
 
DM LECTURE PROJECT.pptx
DM LECTURE  PROJECT.pptxDM LECTURE  PROJECT.pptx
DM LECTURE PROJECT.pptx
 
Learning case of strongly positive PET-CT Rubidium 82 on all parameters.ppt
Learning case of strongly positive PET-CT Rubidium 82 on all parameters.pptLearning case of strongly positive PET-CT Rubidium 82 on all parameters.ppt
Learning case of strongly positive PET-CT Rubidium 82 on all parameters.ppt
 
The Role of PET-CT imaging in the diagnosis of cardiac sarcoidosis post cardi...
The Role of PET-CT imaging in the diagnosis of cardiac sarcoidosis post cardi...The Role of PET-CT imaging in the diagnosis of cardiac sarcoidosis post cardi...
The Role of PET-CT imaging in the diagnosis of cardiac sarcoidosis post cardi...
 
Diabetes mellitus and vascular disease 2022 FINALD.pptx
Diabetes mellitus and vascular disease 2022 FINALD.pptxDiabetes mellitus and vascular disease 2022 FINALD.pptx
Diabetes mellitus and vascular disease 2022 FINALD.pptx
 
Nuc part BBBB.pptx
Nuc part BBBB.pptxNuc part BBBB.pptx
Nuc part BBBB.pptx
 
Non Invasive testing of myocardial ischemia AA.pptx
Non Invasive testing of myocardial ischemia AA.pptxNon Invasive testing of myocardial ischemia AA.pptx
Non Invasive testing of myocardial ischemia AA.pptx
 
Inherited aortopathy2022
Inherited aortopathy2022Inherited aortopathy2022
Inherited aortopathy2022
 
The role of cardiac devise therapy in heart2021 ihab
The role of cardiac devise therapy in heart2021 ihabThe role of cardiac devise therapy in heart2021 ihab
The role of cardiac devise therapy in heart2021 ihab
 

Último

Call Girls Bhubaneswar Just Call 9907093804 Top Class Call Girl Service Avail...
Call Girls Bhubaneswar Just Call 9907093804 Top Class Call Girl Service Avail...Call Girls Bhubaneswar Just Call 9907093804 Top Class Call Girl Service Avail...
Call Girls Bhubaneswar Just Call 9907093804 Top Class Call Girl Service Avail...Dipal Arora
 
Call Girls Varanasi Just Call 9907093804 Top Class Call Girl Service Available
Call Girls Varanasi Just Call 9907093804 Top Class Call Girl Service AvailableCall Girls Varanasi Just Call 9907093804 Top Class Call Girl Service Available
Call Girls Varanasi Just Call 9907093804 Top Class Call Girl Service AvailableDipal Arora
 
Premium Bangalore Call Girls Jigani Dail 6378878445 Escort Service For Hot Ma...
Premium Bangalore Call Girls Jigani Dail 6378878445 Escort Service For Hot Ma...Premium Bangalore Call Girls Jigani Dail 6378878445 Escort Service For Hot Ma...
Premium Bangalore Call Girls Jigani Dail 6378878445 Escort Service For Hot Ma...tanya dube
 
Call Girls Jabalpur Just Call 9907093804 Top Class Call Girl Service Available
Call Girls Jabalpur Just Call 9907093804 Top Class Call Girl Service AvailableCall Girls Jabalpur Just Call 9907093804 Top Class Call Girl Service Available
Call Girls Jabalpur Just Call 9907093804 Top Class Call Girl Service AvailableDipal Arora
 
Call Girls Kochi Just Call 9907093804 Top Class Call Girl Service Available
Call Girls Kochi Just Call 9907093804 Top Class Call Girl Service AvailableCall Girls Kochi Just Call 9907093804 Top Class Call Girl Service Available
Call Girls Kochi Just Call 9907093804 Top Class Call Girl Service AvailableDipal Arora
 
VIP Call Girls Indore Kirti 💚😋 9256729539 🚀 Indore Escorts
VIP Call Girls Indore Kirti 💚😋  9256729539 🚀 Indore EscortsVIP Call Girls Indore Kirti 💚😋  9256729539 🚀 Indore Escorts
VIP Call Girls Indore Kirti 💚😋 9256729539 🚀 Indore Escortsaditipandeya
 
Call Girls Visakhapatnam Just Call 9907093804 Top Class Call Girl Service Ava...
Call Girls Visakhapatnam Just Call 9907093804 Top Class Call Girl Service Ava...Call Girls Visakhapatnam Just Call 9907093804 Top Class Call Girl Service Ava...
Call Girls Visakhapatnam Just Call 9907093804 Top Class Call Girl Service Ava...Dipal Arora
 
Night 7k to 12k Navi Mumbai Call Girl Photo 👉 BOOK NOW 9833363713 👈 ♀️ night ...
Night 7k to 12k Navi Mumbai Call Girl Photo 👉 BOOK NOW 9833363713 👈 ♀️ night ...Night 7k to 12k Navi Mumbai Call Girl Photo 👉 BOOK NOW 9833363713 👈 ♀️ night ...
Night 7k to 12k Navi Mumbai Call Girl Photo 👉 BOOK NOW 9833363713 👈 ♀️ night ...aartirawatdelhi
 
Call Girls Tirupati Just Call 9907093804 Top Class Call Girl Service Available
Call Girls Tirupati Just Call 9907093804 Top Class Call Girl Service AvailableCall Girls Tirupati Just Call 9907093804 Top Class Call Girl Service Available
Call Girls Tirupati Just Call 9907093804 Top Class Call Girl Service AvailableDipal Arora
 
Call Girls Faridabad Just Call 9907093804 Top Class Call Girl Service Available
Call Girls Faridabad Just Call 9907093804 Top Class Call Girl Service AvailableCall Girls Faridabad Just Call 9907093804 Top Class Call Girl Service Available
Call Girls Faridabad Just Call 9907093804 Top Class Call Girl Service AvailableDipal Arora
 
Lucknow Call girls - 8800925952 - 24x7 service with hotel room
Lucknow Call girls - 8800925952 - 24x7 service with hotel roomLucknow Call girls - 8800925952 - 24x7 service with hotel room
Lucknow Call girls - 8800925952 - 24x7 service with hotel roomdiscovermytutordmt
 
Call Girls Ooty Just Call 9907093804 Top Class Call Girl Service Available
Call Girls Ooty Just Call 9907093804 Top Class Call Girl Service AvailableCall Girls Ooty Just Call 9907093804 Top Class Call Girl Service Available
Call Girls Ooty Just Call 9907093804 Top Class Call Girl Service AvailableDipal Arora
 
Call Girls Cuttack Just Call 9907093804 Top Class Call Girl Service Available
Call Girls Cuttack Just Call 9907093804 Top Class Call Girl Service AvailableCall Girls Cuttack Just Call 9907093804 Top Class Call Girl Service Available
Call Girls Cuttack Just Call 9907093804 Top Class Call Girl Service AvailableDipal Arora
 
Night 7k to 12k Chennai City Center Call Girls 👉👉 7427069034⭐⭐ 100% Genuine E...
Night 7k to 12k Chennai City Center Call Girls 👉👉 7427069034⭐⭐ 100% Genuine E...Night 7k to 12k Chennai City Center Call Girls 👉👉 7427069034⭐⭐ 100% Genuine E...
Night 7k to 12k Chennai City Center Call Girls 👉👉 7427069034⭐⭐ 100% Genuine E...hotbabesbook
 
Call Girls Coimbatore Just Call 9907093804 Top Class Call Girl Service Available
Call Girls Coimbatore Just Call 9907093804 Top Class Call Girl Service AvailableCall Girls Coimbatore Just Call 9907093804 Top Class Call Girl Service Available
Call Girls Coimbatore Just Call 9907093804 Top Class Call Girl Service AvailableDipal Arora
 
Best Rate (Hyderabad) Call Girls Jahanuma ⟟ 8250192130 ⟟ High Class Call Girl...
Best Rate (Hyderabad) Call Girls Jahanuma ⟟ 8250192130 ⟟ High Class Call Girl...Best Rate (Hyderabad) Call Girls Jahanuma ⟟ 8250192130 ⟟ High Class Call Girl...
Best Rate (Hyderabad) Call Girls Jahanuma ⟟ 8250192130 ⟟ High Class Call Girl...astropune
 
Call Girls Gwalior Just Call 8617370543 Top Class Call Girl Service Available
Call Girls Gwalior Just Call 8617370543 Top Class Call Girl Service AvailableCall Girls Gwalior Just Call 8617370543 Top Class Call Girl Service Available
Call Girls Gwalior Just Call 8617370543 Top Class Call Girl Service AvailableDipal Arora
 
Best Rate (Patna ) Call Girls Patna ⟟ 8617370543 ⟟ High Class Call Girl In 5 ...
Best Rate (Patna ) Call Girls Patna ⟟ 8617370543 ⟟ High Class Call Girl In 5 ...Best Rate (Patna ) Call Girls Patna ⟟ 8617370543 ⟟ High Class Call Girl In 5 ...
Best Rate (Patna ) Call Girls Patna ⟟ 8617370543 ⟟ High Class Call Girl In 5 ...Dipal Arora
 
(👑VVIP ISHAAN ) Russian Call Girls Service Navi Mumbai🖕9920874524🖕Independent...
(👑VVIP ISHAAN ) Russian Call Girls Service Navi Mumbai🖕9920874524🖕Independent...(👑VVIP ISHAAN ) Russian Call Girls Service Navi Mumbai🖕9920874524🖕Independent...
(👑VVIP ISHAAN ) Russian Call Girls Service Navi Mumbai🖕9920874524🖕Independent...Taniya Sharma
 
All Time Service Available Call Girls Marine Drive 📳 9820252231 For 18+ VIP C...
All Time Service Available Call Girls Marine Drive 📳 9820252231 For 18+ VIP C...All Time Service Available Call Girls Marine Drive 📳 9820252231 For 18+ VIP C...
All Time Service Available Call Girls Marine Drive 📳 9820252231 For 18+ VIP C...Arohi Goyal
 

Último (20)

Call Girls Bhubaneswar Just Call 9907093804 Top Class Call Girl Service Avail...
Call Girls Bhubaneswar Just Call 9907093804 Top Class Call Girl Service Avail...Call Girls Bhubaneswar Just Call 9907093804 Top Class Call Girl Service Avail...
Call Girls Bhubaneswar Just Call 9907093804 Top Class Call Girl Service Avail...
 
Call Girls Varanasi Just Call 9907093804 Top Class Call Girl Service Available
Call Girls Varanasi Just Call 9907093804 Top Class Call Girl Service AvailableCall Girls Varanasi Just Call 9907093804 Top Class Call Girl Service Available
Call Girls Varanasi Just Call 9907093804 Top Class Call Girl Service Available
 
Premium Bangalore Call Girls Jigani Dail 6378878445 Escort Service For Hot Ma...
Premium Bangalore Call Girls Jigani Dail 6378878445 Escort Service For Hot Ma...Premium Bangalore Call Girls Jigani Dail 6378878445 Escort Service For Hot Ma...
Premium Bangalore Call Girls Jigani Dail 6378878445 Escort Service For Hot Ma...
 
Call Girls Jabalpur Just Call 9907093804 Top Class Call Girl Service Available
Call Girls Jabalpur Just Call 9907093804 Top Class Call Girl Service AvailableCall Girls Jabalpur Just Call 9907093804 Top Class Call Girl Service Available
Call Girls Jabalpur Just Call 9907093804 Top Class Call Girl Service Available
 
Call Girls Kochi Just Call 9907093804 Top Class Call Girl Service Available
Call Girls Kochi Just Call 9907093804 Top Class Call Girl Service AvailableCall Girls Kochi Just Call 9907093804 Top Class Call Girl Service Available
Call Girls Kochi Just Call 9907093804 Top Class Call Girl Service Available
 
VIP Call Girls Indore Kirti 💚😋 9256729539 🚀 Indore Escorts
VIP Call Girls Indore Kirti 💚😋  9256729539 🚀 Indore EscortsVIP Call Girls Indore Kirti 💚😋  9256729539 🚀 Indore Escorts
VIP Call Girls Indore Kirti 💚😋 9256729539 🚀 Indore Escorts
 
Call Girls Visakhapatnam Just Call 9907093804 Top Class Call Girl Service Ava...
Call Girls Visakhapatnam Just Call 9907093804 Top Class Call Girl Service Ava...Call Girls Visakhapatnam Just Call 9907093804 Top Class Call Girl Service Ava...
Call Girls Visakhapatnam Just Call 9907093804 Top Class Call Girl Service Ava...
 
Night 7k to 12k Navi Mumbai Call Girl Photo 👉 BOOK NOW 9833363713 👈 ♀️ night ...
Night 7k to 12k Navi Mumbai Call Girl Photo 👉 BOOK NOW 9833363713 👈 ♀️ night ...Night 7k to 12k Navi Mumbai Call Girl Photo 👉 BOOK NOW 9833363713 👈 ♀️ night ...
Night 7k to 12k Navi Mumbai Call Girl Photo 👉 BOOK NOW 9833363713 👈 ♀️ night ...
 
Call Girls Tirupati Just Call 9907093804 Top Class Call Girl Service Available
Call Girls Tirupati Just Call 9907093804 Top Class Call Girl Service AvailableCall Girls Tirupati Just Call 9907093804 Top Class Call Girl Service Available
Call Girls Tirupati Just Call 9907093804 Top Class Call Girl Service Available
 
Call Girls Faridabad Just Call 9907093804 Top Class Call Girl Service Available
Call Girls Faridabad Just Call 9907093804 Top Class Call Girl Service AvailableCall Girls Faridabad Just Call 9907093804 Top Class Call Girl Service Available
Call Girls Faridabad Just Call 9907093804 Top Class Call Girl Service Available
 
Lucknow Call girls - 8800925952 - 24x7 service with hotel room
Lucknow Call girls - 8800925952 - 24x7 service with hotel roomLucknow Call girls - 8800925952 - 24x7 service with hotel room
Lucknow Call girls - 8800925952 - 24x7 service with hotel room
 
Call Girls Ooty Just Call 9907093804 Top Class Call Girl Service Available
Call Girls Ooty Just Call 9907093804 Top Class Call Girl Service AvailableCall Girls Ooty Just Call 9907093804 Top Class Call Girl Service Available
Call Girls Ooty Just Call 9907093804 Top Class Call Girl Service Available
 
Call Girls Cuttack Just Call 9907093804 Top Class Call Girl Service Available
Call Girls Cuttack Just Call 9907093804 Top Class Call Girl Service AvailableCall Girls Cuttack Just Call 9907093804 Top Class Call Girl Service Available
Call Girls Cuttack Just Call 9907093804 Top Class Call Girl Service Available
 
Night 7k to 12k Chennai City Center Call Girls 👉👉 7427069034⭐⭐ 100% Genuine E...
Night 7k to 12k Chennai City Center Call Girls 👉👉 7427069034⭐⭐ 100% Genuine E...Night 7k to 12k Chennai City Center Call Girls 👉👉 7427069034⭐⭐ 100% Genuine E...
Night 7k to 12k Chennai City Center Call Girls 👉👉 7427069034⭐⭐ 100% Genuine E...
 
Call Girls Coimbatore Just Call 9907093804 Top Class Call Girl Service Available
Call Girls Coimbatore Just Call 9907093804 Top Class Call Girl Service AvailableCall Girls Coimbatore Just Call 9907093804 Top Class Call Girl Service Available
Call Girls Coimbatore Just Call 9907093804 Top Class Call Girl Service Available
 
Best Rate (Hyderabad) Call Girls Jahanuma ⟟ 8250192130 ⟟ High Class Call Girl...
Best Rate (Hyderabad) Call Girls Jahanuma ⟟ 8250192130 ⟟ High Class Call Girl...Best Rate (Hyderabad) Call Girls Jahanuma ⟟ 8250192130 ⟟ High Class Call Girl...
Best Rate (Hyderabad) Call Girls Jahanuma ⟟ 8250192130 ⟟ High Class Call Girl...
 
Call Girls Gwalior Just Call 8617370543 Top Class Call Girl Service Available
Call Girls Gwalior Just Call 8617370543 Top Class Call Girl Service AvailableCall Girls Gwalior Just Call 8617370543 Top Class Call Girl Service Available
Call Girls Gwalior Just Call 8617370543 Top Class Call Girl Service Available
 
Best Rate (Patna ) Call Girls Patna ⟟ 8617370543 ⟟ High Class Call Girl In 5 ...
Best Rate (Patna ) Call Girls Patna ⟟ 8617370543 ⟟ High Class Call Girl In 5 ...Best Rate (Patna ) Call Girls Patna ⟟ 8617370543 ⟟ High Class Call Girl In 5 ...
Best Rate (Patna ) Call Girls Patna ⟟ 8617370543 ⟟ High Class Call Girl In 5 ...
 
(👑VVIP ISHAAN ) Russian Call Girls Service Navi Mumbai🖕9920874524🖕Independent...
(👑VVIP ISHAAN ) Russian Call Girls Service Navi Mumbai🖕9920874524🖕Independent...(👑VVIP ISHAAN ) Russian Call Girls Service Navi Mumbai🖕9920874524🖕Independent...
(👑VVIP ISHAAN ) Russian Call Girls Service Navi Mumbai🖕9920874524🖕Independent...
 
All Time Service Available Call Girls Marine Drive 📳 9820252231 For 18+ VIP C...
All Time Service Available Call Girls Marine Drive 📳 9820252231 For 18+ VIP C...All Time Service Available Call Girls Marine Drive 📳 9820252231 For 18+ VIP C...
All Time Service Available Call Girls Marine Drive 📳 9820252231 For 18+ VIP C...
 

Cardiology Lecture to IM Board Exam oriented MCQS Picture Quiz Explanatory notes

  • 1. Cardiology Lecture to IM Board Exam oriented MCQS Picture Quiz Explanatory notes Dr Ihab Suliman October 2013
  • 3.
  • 4.
  • 6. Introductory MCQs       The best BB associated with significant & proven Mortality benefit post MI ? a- Atenolol b- Propranolol c- Timolol d- carvedilol (Dilatrend) e-Metoprolol tartrate, Short acting
  • 7.  d- carvedilol (Dilatrend)  Other Medications with mortality benefit post MI are Metoprolol succinate , bisoprolol ( Concor), ACEI, ARBs , Spironolactone, statins 
  • 8. Q 23       A 56 year old obese female presents for a routine physical examination. Her lipid profile  reveals a significantly elevated triglyceride level of 355 mg/dL. Which of the following medications can act to lower her triglyceride level by stimulating the synthesis of lipoprotein lipase? A) gemfibrozil B) rosuvastatin C) cholestyramine D) ezetimibe E) ketoconazole
  • 9. A 23  A) gemfibrozil  Gemfibrozil acts by stimulating the synthesis of lipoprotein lipase to degrade triglycerides  into fatty acids increasing their metabolism and lowering blood levels. Elevated triglyceride levels can lead to atherosclerosis and coronary artery disease. Rosuvastatin (B) is an HMG-CoA reductase inhibitor which can cause rhabdomyolysis or hepatic dysfunction (elevation in AST and ALT levels). Cholestyramine (C) is a bile acid binding resin used to treat elevated lowdensity lipoprotein levels (LDL). Ezetimibe (D) is also used to treat elevated LDL levels and acts by inhibiting cholesterol absorption at the brush border of the small intestine. Ketoconazole (E), an anti-fungal medication, significantly reduces LDL levels as well.  
  • 10. Q1  A 19 year old boy with no past medical history passes out while running at a school evnt. He states that he was feeling a little dizzy prior to the event, but does not complains of chest pain or palpitations. His blood pressure is 120/85, heart rate 85/min, Respiratory Rate 12/min, and he is afebrile. His physical examination reveals normal lung sounds, a II/VI mid-sytolic creshendo-decreshndo murmur is heard at the right upper sternal border which increases in intensity with Valsalva, an S4 heart sound is also present. Laboratory studies are normal. What is the most appropriate treatment at this time?
  • 11.      A) Start a Diuretic  B) Permanent Pacemaker Implantation  C) AICD Implantation  D) Observation  E) Start  Beta Blocker
  • 13.
  • 14.
  • 15.
  • 16.   The commonest cardiovascular manifestation of the Coxsackie infective agent is a) cardiac tamponade b) congenital Pericardial Defects c) acute viral Pericarditis d) pericardial Cysts e) bronchogenic carcinoma
  • 18.   35 year old obese women develops painful leg on the 10th post operative day following emergency surgery for a ruptured ectopic pregnancy. She is likely to have ? a) thromboangitis obliterans  b) ruptured baker`s cyst  c) deep vein thrombosis of the lower limb  d) aortoiliac artery artherosclerosis  e) embolus
  • 19.   c) deep vein thrombosis of the lower limb  Specially if she is homozygous or heterozygous for coagulopathy factor ?
  • 20.   A 56 year old gardener complains of sudden pain with swelling behind the right knee. He also complains of associated pain and swelling in the calf. On examination a crescentic hematoma is noted below the medial malleolus of the ankle. a) Acute hemarthrosis  b) Osteonecrosis  c) Baker’s cyst  d) pigmented villonodular synovitis  e) rheumatoid arthritis
  • 21.  A Baker cyst is sometimes called a popliteal cyst. When an excess of knee joint fluid is compressed by the body weight between the bones of the knee joint, it can become trapped and separate from the joint to form the fluidfilled sac of a Baker cyst. The name of the cyst is in memory of the physician who originally described the condition, the British surgeon William Morrant Baker (1839-1896).
  • 23.   Mental retardation of varying severity is also common.  Seizures in association with neuronal migration disorders like pachygyria and heterotopias are occasionally seen. Autism may be seen in up to 10% of the patients. Cardiac defects include ventricular septal defects;  renal abnormalities comprise of unilateral renal agenesis and horseshoe kidneys. 
  • 24. Question 1      All of the following is correct about Atrial Fibrillation except ? A- The most common persistent arrhythmia. B- Incidence or prevalence increases with age. C- if hemodynamically unstable patient in ER should be cardioverted . D- Dabigatran( Pradaxa) new antithrombin associated with Decresed GI bleed compared to Warfarin
  • 25.    D- Dabigatran new antithrombin associated with Decresed GI bleed compared to Warfarin The correct answer is Dabigatran new antithrombin associated with INCREASED GI bleed compared to Warfarin
  • 26. Atrial fibrillation is a common arrythmia • In the FHS, the lifetime risk of atrial fibrillation (AFib) for adults age 40 is 26% for men and 23% for women1 • Prevalence of AFib in the US is estimated > 2.2 million adults with an incidence > 70 000 2 • Prevalence increases steeply with age 2 1.Lloyd-Jones DM, et al. Circulation 2004;110:1042– 1046
  • 27. AFib: a common but serious arrythmia  AFib increases the risk of stroke 5-fold1  The increase in risk of stroke is similar for paroxysmal, persistent and permanent AFib2  Strokes associated with AFib are usually more severe than those from other causes, conferring an increased risk of morbidity, mortality and poor functional outcome1 1. Savelieva et al. Ann Med 2007;39:371– 2. Hart R et al. JACC 2000; 35:183
  • 28. Dabigatran etexilate – a new, reversible, oral DTI        Dabigatran etexilate is an oral prodrug, converted to dabigatran, a potent reversible DTI No interaction with food No participation with CYP450 Predictable anticoagulant effect Fixed dose No need for routine coagulation monitoring Very recently some genetic variation for response 28
  • 29. Q4   A 68-year-old gentleman is admitted with an Anterior myocardial infarction (MI) and receives thrombolysis, aspirin, atenolol, atorvastatin and lisinopril. His ECG shows good ST segment resolution. The next day he develops some pain in the legs and a dusky discolouration of the lower limbs. On closer examination there is a diffuse petechial rash over the lower limbs, particularly the feet, but all peripheral pulses are palpable.  CBC showed elevated eosinophils
  • 30. Q4       Which of the following is the most likely cause for his current situation? A)Polyarteritis Nodosa B)Aspirin Allergy C)Periphral Vascular Disease D)Cholesterol Emboli E)Post Thrombolysis Allergy
  • 31. A4 D)Cholesterol Emboli The  above patient is an arteriopath as suggested by the acute Myocardial Infarction, and one day after thrombolysis he develops a petechial rash in the lower limbs with raised white cell count marked eosinophilia and raised IgE. This suggests cholesterol embolisation syndrome rather than allergy.  
  • 32. Question 2  Catheter-delivered balloon expansion techniques are now the treatment of choice for which one of the following lesions in adults?  A. Valvular pulmonic stenosis.  B. Valvular aortic stenosis.  C. Coarctation of the aorta.  D. Ebstein's anomaly of the tricuspid valve.  E. Severe mitral stenosis with Significant regurgitation.
  • 33. Answer to Q 2  Comment The correct answer is A.  Although catheter balloon valvuloplasty and aortoplasty have been attempted in all these conditions, only pulmonary valvotomy has achieved a success level consistent with being the treatment of choice in adults. Aortic stenosis responds initially to balloon expansion and may serve as a bridge to valve replacement surgery, but is associated with rapid restenosis. Success rates with coarctation and Ebstein's anomaly are not uniform enough to displace surgery except in selected patients. Mitral stenosis in the absence of severe subvalvular disease can be successfully treated by balloon valvuloplasty, but the presence of moderate to severe regurgitation is an indication for surgery.
  • 34. Q7  A 72 year old female with a history of breast cancer and tobacco use complains of dizziness and dyspnea on exertion. Her heart sounds are distant and her systolic blood pressure is noted to markedly decrease with inspiration. Which of the following is the most likely diagnosis?  A) Constrictive cardiomyopathy B) Mitral valve Stenosis C) Congestive heart failure D) Pulmonary embolus E) Cardiac Tamponade    
  • 35. A7  E) Cardiac Tamponade      Cancer is the most common cause of pericardial effusion and when enough fluid accumulates in the pericardial space, cardiac tamponade occurs. “Pulsus paradoxus” is when there is a decrease in systolic blood pressure during inspiration due to failure of the right ventricle to accept the normal increased venous return that occurs with inspiration. This also results in a “Kussmal’s sign” or elevated jugular venous distension during inspiration (normally the opposite occurs). Treatment is with emergent pericardiocentesis.
  • 36. Q3  In which of the following diseases is pregnancy difficult, but not highly risky to mother and fetus?  A. Eisenmenger's syndrome.  B. Primary pulmonary hypertension.  C. Hypertrophic obstructive cardiomyopathy.  D. Prior peripartum cardiomyopathy with heart failure.  E. The Marfan syndrome with dilated aortic root.
  • 37. Answer to Q3   Comment The correct answer is C. The cardiovascular system must be able to handle a doubling of cardiac output during pregnancy. Thus, cardiopulmonary diseases that obstruct blood flow are usually contraindications to pregnancy because both the mother and fetus get inadequate blood flow. Thus, obstruction to pulmonary flow due to the Eisenmenger reaction or primary pulmonary hypertension fits into this category, but hypertrophic cardiomyopathy does not. The increased cardiac output increases venous return to the left heart resulting in left ventricular enlargement and less obstruction. In fact, during pregnancy the murmur of hypertrophic obstructive cardiomyopathy may lessen or even disappear, causing the diagnosis to be missed. Prior peripartum cardiomyopathy with heart failure is a contraindication to pregnancy because of the high incidence of recurrent failure and death. Hormonal changes during pregnancy alter vascular walls, making them more distensible. This is a normal mechanism to adapt to higher cardiac output; however, in the patient with the Marfan syndrome and an enlarged aortic root, it can lead to increased wall stress and aortic rupture or dissection.
  • 38. Q8  A 50 year old gentleman is admitted to the CCU with an acute Inferior wall infarction. Three hours after the admission his B.P is 90/50 mmHg. The heart rate is 38 beats per minute with sinus rhythm. Which of the following will be the most appropriate initial therapy?  A)Immediate insertion of temporary pacemaker B)Intravenous administration of of Isoproterenol 5 micrograms/min C)Intravenous administration of Dobutamine 0.35 mg/min D) Administration of Intravenous Normal saline, 300 ml over 15 mins E) Intravenous administration of atropine sulfate , 0.6 mg    
  • 39. A8  E) Intravenous administration of atropine sulfate , 0.6 mg
  • 40. Q9  A 50 year old gentleman with a history of hypertension and high cholesterol presents to the emergency department with pain in chest for one hour. He describes a substernal chest pressure associated with shortness of breath and sweating. His ECG shows ST elevations consistent with myocardial infarction. Which of the following laboratory results would be expected?  A) Elevated myoglobin, normal troponin I, and normal CK-MB  B)Normal myoglobin, elevated troponin I, and normal CK-MB  C)Normal myoglobin, normal troponin I, and elevated CK-MB  D)Normal myoglobin, normal troponin I, and normal CK-MB   
  • 41. A9    A) Elevated myoglobin, normal troponin I, and normal CK-MB Myoglobin is a heme protein found in skeletal and cardiac muscle that has attracted considerable interest as an early marker of MI. Its low molecular weight accounts for its early release profile: myoglobin typically rises1- 2hours after onset of infarction, peaks at 6-12 hours, and returns to normal within 24-36 hours.
  • 42.
  • 43. Q10  A 60 year old man with a histoty of Diabetes, hypertension and congestive heart failure was brought to the ER after he complained of blurred vision and headache. He was found to have Blood Pressure of 220/90 mm Hg. The intern who examined her wanted to give her drug X, but the attending doctor rejects this choice because of its tendency to cause compensatory tachycardia and exacerbate fluid retention. The drug can also cause lupus like syndrome with long term use. What is the mechanism of action of Drug X?  A)Blocks Calcium Channels  B)Increases Production of cGMP  C)Decreases Production of cGMP  D)Inhibits Angiotensin Converting Enzyme  e)Inhibits carbonic Anhydrase
  • 44. A 10     Hydralazine increases  (cGMP) levels, increasing the activity of protein kinase G (PKG). Active PKG adds an inhibitory phosphate to myosin light-chain kinase (MLCK) – a protein involved in the activation of cross-bridge cycling (i.e. contraction) in smooth muscle. This results in blood vessel relaxation. It dilates arterioles more than veins. Hydralazine requires the endothelium to provide NO) thus only provides the effects of NO in vivo with functional endothelium. It will not work to vasodilate in vitro in an isolated blood vessel. Activation of  has been suggested as a mechanism
  • 45. Q 11  A 46 year old gentleman came to the Emergency department with c/o sudden dyspnea. Patient had similar episode of dyspnea two days earlier which was associated with profuse sweating. Patient is a reformed smoker and a social drinker. ECG is displayed below. Let`s interpret the ECG and decide the next step in the management.
  • 46. Sinus; PVCs; LAD; anterior symmetric T wave inversion - and almost "Wellens"-like in V2,V3 - so patient will probably need cath in short order (looking for a tight LAD lesion ...).
  • 47. Q 12  A 30 year old female with no significant past medical history is 42 weeks pregnant and labor has just begun. She begins to complain of difficulty in breathing which worsens throughout delivery to the point of requiring intubation. She had never previously complained of any shortness of breath or chest pain. Her temperature is 37.0, blood pressure 90/60, heart rate 135, respirations 25, and oxygen saturation 100% on 60% FiO2 on the ventilator. Physical examination reveals diffuse pulmonary rales, a II/IV early diastolic murmur, and no lower extremity edema. Laboratory studies are normal. ECG shows sinus tachycardia and left atrial enlargement. Her chest x-ray has pulmonary edema. Which of the following is the most likely diagnosis?  A) Aortic Dissection B)Dissection of Coronary Artery C)Aortic valve Stenosis D) Mitral Valve Stenosis   
  • 49. Q 13  A 60 year old gentleman with a history of dyslipidemia, hypertension and CHF presents to the emergency department with acute onset swelling of the lips, eyes, and tongue causing airway obstruction and respiratory distress. He also has an urticarial rash. Which of the following medications is the likely culprit of his current symptoms?   A)Pindolol  B)Clonidine  C)Felodipine  D)Captopril  E)Methyldopa    
  • 50.
  • 51.
  • 52.
  • 54.
  • 55. Butter fly skin rash of SLE
  • 56. Q 14       A 72 year old female with no significant past medical history passes out while exercising. She has intermittent exertional chest pains and dyspnea on exertion as well. Her physical examination reveals a III/VI late-peaking creshendo-decreshendo murmur at the right upper sternal border and a III/VI holosystolic murmur at the apex. Her S2 heart sound is very soft and her carotid upstroke is weak and delayed. Which of the following is most likely causing her symptoms? A) Aortic valve regurgitation B) Aortic valve stenosis C) Mitral valve regurgitation D) Mitral valve stenosis E) Mitral valve prolapse
  • 57. A 14  Aortic valve stenosis (AS) presents with one of the classic triad: syncope (passing out), exertional angina, or exertional dyspnea (from heart failure).  Over the age of 70 the most likely cause is degenerative calcific aortic stenosis while under the age of 70 a bicuspid aortic valve is the likely culprit. Rheumatic heart disease is the third leading cause. Physical examination reveals a crescendo-decrescendo murmur at the aortic listening post (right upper sternal border) which radiates to the carotids. The more severe the aortic stenosis the later the peak of the murmur in systole and the softer the A2 component of the S2 heart sound. The murmur is depicted below:  “Pulses parvus et tardus” is present upon carotid artery examination. Remember parvus means weak and tardus means late. The murmur can radiate to the apex and sound holosystolic mimicking mitral regurgitation (this is known as the Galiverdin phenomenon). No medical treatment is available for aortic stenosis. Aortic valve replacement surgically is indicated for anyone who is symptomatic.
  • 58. Q 15       A 56 year old African American female with a history of asthma is diagnosed with hypertension. Laboratory studies reveal a creatinine of 3.0 mg/dL and a potassium level of 5.1 mg/dL. Which of the following medications is appropriate to treat her hypertension? A) amlodipine B) hydrochlorothiazide C) enalapril D) propranolol E) spironolactone
  • 59. Calcium channel blockers        Mechanism of Action Calcium channel blockers (CCB) are a group of medications that act to inhibit either central or peripheral calcium channels. This results in decreased calcium influx and smooth muscle relaxation. Dihydropyridine calcium channel blockers act predominantly in the periphery causing arterial vasodilation. Non-dihydropyridine calcium channel blockers act predominantly centrally to decreased heart rate (chronotropy) and contractility (inotropy). Indications Calcium channel blockers are used to treat hypertension and stable angina. The non-dihydropyridine calcium channel blockers are used to lower heart rate during tachyarrhythmias especially atrial fibrillation and atrial flutter as well as during acute coronary syndromes when beta-blockers are contraindicated. Verapamil has been studied to treat hypertrophic obstructive cardiomyopathy as well as multifocal atrial tachycardia.
  • 60. Q 16       A 29 year old male with a history of ectopia lentis presents for a routine physical examination. He is noted to have pectus excavatum, pes planus, a high arched palate, and a positive wrist and thumb sign. Which of the following cardiac disorders is associated with his condition? A) Aortic valve stenosis B) Coarctation of the aorta C) Mitral valve prolapse D) Ventricular septal defect E) Ebstein’s anomaly
  • 61. A 16    C) Mitral valve prolapse Marfan’s syndrome is associated with mitral valve prolapse and aortic aneurysms. Specifically, the ascending aorta may dilated and predispose patient’s to acute aortic dissection which can be fatal. Also, when the ascending aorta dilates, the aortic valve annulus stretches causing the valve leaflets to fail to coapt which results in aortic regurgitation. Aortic stenosis (A) is not associated with Marfan’s syndrome and is caused be either senile calcific degeneration of the valve or from a congenital bicuspid aortic valve . Coartaction of the aorta (B) is associated with Turner’s syndrome and presents with hypertension in the upper extremities and hypotension in the lower extremities. “Rib notching” is seen on the chest xray.  Ventricular septal defects  (D) and Ebstein’s anomaly  (E) are not associated with Marfan’s. 
  • 62. Q4  In long-term follow-up of patients after surgical repair of tetralogy of Fallot, the most common dysrhythmia observed is:  A. Sinus bradycardia.  B. Atrial fibrillation.  C. Atrial tachycardia.  D. Ventricular tachycardia.  E. Junctional tachycardia.
  • 63. Answer Q 4  Comment The correct answer is D.  Complex ventricular arrhythmias often occur during long-term follow-up of patients with tetralogy of Fallot. The incidence correlates with age at repair and with higher residual postoperative right ventricular systolic and end-diastolic pressures. Sudden death accounts for a significant proportion of the late mortality among these patients. In patients with ventricular tachycardia, the site of origin is typically found to be in the right ventricular outflow tract related to the previous ventriculotomy and infundibular resection.
  • 64. Question 5  A 42-year-old man is referred for evaluation of a systolic murmur. Your exam shows normal carotid pulses, a prominent apical impulse, an early systolic sound, and a grade III/VI mid-systolic murmur at the base. Respiration did not change the character of these auscultatory findings. After an extrasystole, the systolic murmur increased in intensity. Handgrip did not alter the systolic murmur. Valsalva decreased the intensity of the murmur, and it returned to baseline intensity after seven heart beats.  Which one of the following diagnoses is most likely?  A. Congenital pulmonic stenosis.  B. Innocent murmur.  C. Mitral valve prolapse.  D. Hypertrophic obstructive cardiomyopathy.  E. Bicuspid aortic valve.
  • 65. Answer to Q 5  Comment The correct answer is E.  A systolic murmur that increases in intensity in the beat following an extrasystole is usually due to turbulent flow out of the ventricles. Mitral regurgitation is less likely because this murmur does not change following an extrasystole. The murmur of hypertrophic obstructive myopathy usually decreases with handgrip exercise. An innocent flow murmur is less likely because of the presence of an early systolic sound and grade III intensity. With pulmonic stenosis, there are characteristic changes in the intensity of the murmur and the ejection sound during respiration. The ejection sound establishes the diagnosis of an abnormal aortic valve, a bicuspid valve being the most common abnormality.
  • 66. Q6      Regarding the Renin-Angiotensin system, which if the following is false ? a. Chronic renal hypoperfusion leads to catecholamine release, hypertension, cardiac hypertrophy and salt and water retention b. Angiotensin Converting Enzyme (ACE) inhibitors work by inhibiting Angiotensinogen cleavage to Angiotensin I c. ACE-2 is not inhibited by Angiotensin Converting Enzyme (ACE) inhibitors d. Bradykinin is thought to mediate the cough associated with ACE inhibitors
  • 67. Answer to Q 6  b. Angiotensin Converting Enzyme (ACE) inhibitors work by inhibiting Angiotensinogen cleavage to Angiotensin I
  • 68. Q7        Which of the following structures are not found in the right atrium? a. Tendon of Todaro b. Moderator band c. Koch's triangle d. Pectinate muscle c. Koch's triangle d. Pectinate muscle
  • 69. Answer to Q7    b. Moderator band The moderator band, located in the right ventricle, is a muscular bridge that connects the distal septum and the right ventricular free wall at the anterior papillary muscle. The tendon of Todaro is a fibrous band located between the valves of the inferior vena cava and coronary sinus in the right atrium.
  • 70.  Koch's triangle is located in the lower medial portion of the right atrium, overlying the AV node and the proximal His bundle. Pectinate muscles arise from the crista terminalis and course as bands on the right atrial free wall.
  • 71. 32 years old male came with fever and acute CVA Zoomed view at MV
  • 72.  M V vegetations ( endocarditis)
  • 73. Major Criteria  1. Positive blood culture  Typical organism from 2 separate cultures- Viridans streptococci, Strptococcus bovis, HACEK, S. aureus, enterococci OR  Persistently + blood culture- all of three/ majority 4 blood cultures OR  Single +ve blood culture for Coxiella or phase IgG > 1:800
  • 74.  2. Evidence of endocardial involvement ECHO- oscillating intracardiac mass OR  Abscess OR  New partial deheiscence of prosthetic valve/ new regurgitation 
  • 75. Minor criteria      Predisposing heart condition Fever > 100.4F Vascular- Emboli, pulmonary infarct, mycotic aneurysm, Janeway lesions Immune- Osler’s nodes, Roth spots, glomerulonephritis Microbiological evidence
  • 76. Modified Duke Criteria  Definite IE       Microorganism (via culture or histology) in a valvular vegetation, embolized vegetation, or intracardiac abscess Histologic evidence of vegetation or intracardiac abscess 2 Major 1 Major + 3 minor 5 Minor Possible IE   1 major and 1 minor 3 minor
  • 77. Janeway Lesions 1. More specific 2. Erythematous, blanching macules 3. Nonpainful 4. Located on palms and soles 5. Microabscess of the dermis with marked necrosis and inflammatory infiltrate not involving the epidermis.
  • 78. Q8      2. Which of the following statements about the pericardium is false? a. The pericardium contains 80 to 90 mL of pericardial fluid. b. It is connected to the diaphragm, sternum, and pleurae. c. It encloses the entire main pulmonary artery. d. The ligament of Marshall is a pericardial fold that contains the remnant of the embryonic left superior vena cava.
  • 79. Answer Q 8   a. The pericardium contains 80 to 90 mL of pericardial fluid. The Correct answer is 10—50 mls
  • 80. Q 18       A 45 year old female with a history of hypertension previously controlled with diet and lifestyle modifications presents to here primary care physician with increasing headaches. Her blood pressure is 160/90. She is diagnosed with worsening migraine headaches. Which of the following medications is appropriate to treat her hypertension and headaches simultaneously? A) hydrochlorothiazide B) metoprolol C) clonidine D) methyldopa E) lisinopril
  • 81. A 18  Metoprolol is a lipid soluble beta-blocker which can cross the blood-brain barrier easily and have been shown to be effective in the prophylaxis of migraine headaches as well as the treatment of hypertension. They have slight sedating effects and therefore can also be used for stage freight or panic attacks as well.
  • 82. Q9      5. Which of the following statements about the tricuspid valve is true? a. It has three leaflets: anterior, lateral, and posterior. b. It is the largest heart valve. c. It is the most anterior heart valve. d. The posterior leaflet is the largest and most mobile
  • 83. Dimeter of Valves  AV:  TRICUSPID 13 cm  MITRAL 11 cm  SEMILUNAR:  PULMONIC 8 cm  AORTIC 6 cm
  • 84.   Answer is b: The tricuspid valve is the largest heart, whereas the pulmonic valve is the most anterior. The tricuspid valve consists of three leaflets, anterior, septal, and posterior. The anterior leaflet is the largest and most mobile. The posterior leaflet is the smallest, whereas the septal leaflet is the least mobile and occasionally absent.
  • 85. Q 10       1. Which of the following statements about the assessment of the severity of a valvular abnormality are not true? a. Severe acute aortic regurgitation results in a holodiastolic murmur. b. Severe aortic stenosis results in a long-duration and latepeaking systolic ejection murumur. c. An S2-OS interval of less than 70 milliseconds is consistent with severe mitral stenosis with normal heart rates and absence of other left-sided valve disease. d. Severe aortic stenosis may be present with a soft murmur. e. The duration of a tricuspid stenosis murmur correlates with the severity of stenosis
  • 86. Answer to Q 10  The answer is a: The severity of AS is best determined by the duration and time to peak of the murmur. Longduration and late peaking are consistent with severe AS. The intensity of an AS murmur is dependent on the stroke volume, so a soft murmur may occur with severe AS and LV dysfunction. Similarly, the duration of TS or MS murmurs are helpful in determining severity, as is the duration of the S2-OS interval. However, the S2-OS interval is also affected by heart rate and other valvular lesions that affect LA and LV pressure. Severe
  • 87. Q 19       A 67 year old male suffers a large myocardial infarction complicated by sustained ventricular tachycardia. He is started on a lidocaine infusion. He subsequently develops shock liver from hypotension during the ventricular tachycardia and his serum lidocaine levels become significantly elevated. Which of the following is a manifestation of lidocaine toxicity? A) Stroke B) Seizures C) Renal failure D) Congestive heart failure E) Hyperkalemia
  • 88. A 19  B) Seizures  Lidocaine which is used to treat ventricular arrhythmias such as ventricular tachycardia or ventricular fibrillation, can easily reach toxic levels and can cause seizures and may progress to coma and death. Lidocaine at high levels first inhibits the inhibitory neurons in the brain resulting in seizures. Eventually all neurons are inhibited and coma ensues. No specific treatment or antidote exists. 
  • 89. Q 21       A 62 year old female with a history of hypertension presents with increasing shortness of breath, abdominal pain, and diarrhea. Computed tomography reveals a mass in the appendix and multiple nodules in the liver. Chest x-ray is normal. Serum 5hydroxyindoleacetic acid levels are elevated. Which of the following cardiac disorders is she likely to have? A) Aortic valve stenosis B) Aortic valve regurgitation C) Mitral valve stenosis D) Mitral valve regurgitation E) Tricuspid valve stenosis
  • 90. A 21    E) Tricuspid valve stenosis This patient has carcinoid syndrome which consists of diarrhea, facial flushing, reactive airways causing shortness of breath, and cardiac valvular disease specifically of right-sided heart valves since the toxins produced by the tumor are filtered by the lungs and never reach the left sided heart valves (unless pulmonary metastasis are present). Others are all left-sided heart valves which would not be affected in carcinoid syndrome unless pulmonary metastasis are present (which is rare).
  • 91. Q 22      A 29 year old male with no past medical history has been experiencing headaches for the past few months. His blood pressure is noted to be 210/110. Physical examination reveals an S4 heart sound and reduced femoral pulses. Which of the following is associated with his condition? A) Atrial septal defect B) Wolff-Parkinson-White syndrome C) Bicuspid aortic valve D) Mitral valve regurgitation
  • 92. A 22    C) Bicuspid aortic valve Coarctation of the aorta occurs when the congenital narrowing of the aorta occurs. About two thirds of patients with coarctation of the aorta have a bicuspid aortic valve as well. Depending on the location of the narrowing differing presentations may occur. Infantile coarctation of the aorta presents when the stenosis is proximal or next to the ductus arteriosus. When the ductus arteriosus closes (as it should in normal infants), a severe increase in afterload occurs resulting in congestive heart failure (since blood was normally able to traverse the patent ductus arteriosus resulting in lower resistance, then suddenly is unable to).
  • 93. Q 11       Which of the following will not cause a continuous murmur? a. An right coronary sinus to the right atrial fistula. b. A stenotic and regurgitant aortic valve. c. A venous hum. d. An aorta-to-pulmonary artery connection. e. A high-grade coarctation of the aorta.
  • 94. Answer to Q 1 1  The answer is b: A continuous murmur must extend uninterrupted through S2. An aortapulmonary connection such as a PDA, a coronary sinus-to-RA connection such as with a ruptured sinus of Valsalva, a severe coarctation, and a venous hum all result in continuous murmurs. The combination of a systolic and diastolic murmur of AS and AR is not considered a continuous murmur.
  • 95. Q 27       An 82 year old male with a history of long standing atrial fibrillation and hypertension presents with increasing dyspnea on exertion. Chest x-ray shows a honeycoming pattern and pulmonary function testing shows a severe restrictive defect. Which of the following is likely causing his symptoms? A) Congestive heart failure B) amiodarone C) ramipril D) sotalol E) diltiazem
  • 96. A 27        B) amiodarone C) ramipril D) sotalol E) diltiazem Amiodarone used to treat atrial fibrillation and ventricular arrhythmias, can cause pulmonary fibrosis after long-term use. Amiodarone also causes hypothyroidism, hyperthyroidism, and on rare occasion liver failure. Remember to check PFTs (pulmonary function tests), LFTs (liver function tests), and TFTs (thyroid function tests) on all patients on amiodarone. Blue man syndrome can occur as well due to deposition of amiodarone metabolites in the skin resulting in a blue hue.
  • 97. Q 12      Which of the following about differentiating AS from HCM on examination is true? a. The murmur of AS decreases and the murmur of HCM increases with amyl nitrite. b. Post-PVC, the pulse pressure of HCM decreases and that of AS increases. c. The murmur of AS and HCM decrease with standing. d. With Valsalva, the murmur of AS and HCM decrease.
  • 98. Answer to Q 12  The answer is b: Amyl nitrite results in an increase in the murmur of AS and HOCM. Valsalva (straining phase) and standing result in an increase of the murmur of HOCM and decreases the murmur of AS. Post-PVC, the murmur of AS and HOCM increases but the pulse pressure of HOCM decreases (Brockenbrough phenomenon) while that of AS increases.
  • 99. Q 29  A 52 year old female is experiencing difficulty hearing. Which of the following drugs may be the cause?  A) bumetinide  B) clonidine C) minoxidil D) triamterene  
  • 100. A 29   A) bumetinide Ototoxicity occurs with high dose loop diuretics such as furosemide, bumetinide, or torsemide. Aminoglycosides can cause similar hearing loss (also nephrotoxicity). Remember that congenital ear malformations are associated with congenital kidney problems to help remember the connection between medications that act on the kidney and cause hearing loss such as loop diuretics.
  • 101. Q 13       Regarding cardiac myxoma, all of the following statements are true except: a. It can arise anywhere within the heart. b. It is the most common primary cardiac tumor. c. Approximately 75% occur in the left atrium. d. Most are familial. e. Myxomas may be associated with syncope, TIA, or stroke
  • 102. Q 14  Answer is d: Cardiac myxoma is the most common primary cardiac tumor, accounting for 50% of primary cardiac tumors in adults. Though it can be found anywhere in the heart, the most common location is the left atrium attached
  • 103. Q 15       What of the following malignancies has the highest likelihood of cardiac metastasis? a. Lung cancer b. Renal cell carcinoma c. Melanoma d. Breast cancer e. Colon cancer
  • 104. Answer to Q 15  Answer is c: The malignancy with the highest likelihood of cardiac metastasis is melanoma, though in absolute numbers, cardiac metastases are more common, with lung and breast cancers reflecting the higher incidence of these cancers.
  • 105. Q 38      A 55 year old female with a history of congestive heart failure is found to have breast cancer requiring chemotherapy. Which of the following chemotherapeutic agents should be avoided? A) doxorubicin B) bleomycin C) paclitaxel D) cyclophosphamide
  • 106. A 38  A) doxorubicin  Doxorubicin and daunorubicin are anthracycline chemotherapeutic agents that are well known to cause systolic congestive heart failure especially at higher doses and should be avoided if pre-existing heart failure is present. Bleomycin (B) can cause pulmonary fibrosis. Paclitaxel (C) also causes pulmonary toxicity. Cyclophosphamide (D) can cause hemorrhagic cystitis (resulting in hematuria and bladder pain).   
  • 107. Q 16       Digitalis is a cardiac glycoside that does all of the following except: a. indirectly activates a Na+/Ca++ exchanger found in the cardiac cell membrane b. inhibits the Na+/K+ ATPase found in the cardiac cell membrane c. increases chronotropy by increasing intracellular Ca+ + d. leads to a sodium gradient across the cardiac cell membrane that is favorable for Ca++ influx e. increases parasympathetic tone
  • 108. Answer to Q 16   Answer is c: Digitalis inhibits the Na+/K+ ATPase found in the cardiac cell membrane which maintains a high intracellular K+ concentration and high extracellular Na+ concentration. Ca2+ is removed from the cytosol into the extracellular fluid by a sodium-calcium exchange pump driven by the pre-existing Na+ gradient. Inhibiting the Na+/K+ ATPase promotes enhanced Na+/Ca2+ exchange ultimately leading to increased intracellular Ca2+ available to the contractile apparatus increasing myocardial contractility. Chronotropy would not be increased with digitalis and in fact it is often used for the opposite effect of heart rate control in patients with atrial fibrillation.
  • 109. Pic Quiz 45 years old with recent MI, This Skin lesion was noted on his Leg
  • 110. Answer 2 pic quiz  Necrobiosis lipoidica of DM  How is it treated ?
  • 111. Q 17       All of the following are class III recommendations in the treatment of unstable coronary syndromes, except: a. Use of fibrinolytic therapy for non-ST-elevation acute coronary syndromes b. Use of abciximab for conservatively managed high-risk patients who continue to have ischemic symptoms c. The use of a low-molecular-weight heparin instead of unfractionated heparin for conservatively managed unstable coronary syndromes d. Use of nitroglycerin within 24 hours of sildenafil (Viagra) e. Invasive therapy in low-risk patients who present with a chest pain syndrome
  • 112. Answer to Q 17  Answer is c: There may be a marginal benefit of low-molecularweight heparin over unfractionated heparin for conservatively managed patients, and this strategy is a class IIa recommendation. Nitroglycerin should not be used within 24 hours from the last dose of sildenafil. Fibrinolytics should only be used for ST-elevation myocardial infarctions. Ideally, high-risk patients should be managed invasively, but for high-risk individuals who defer invasive therapy or who have extensive comorbidities and continue to have ischemic symptoms, the use of a glycoprotein IIb/IIIa inhibitor is a class IIa recommdation. However, eptifibitide or tirofiban should be used in this setting, while abciximab should be used only during invasive management.
  • 113. Q18       Which of the following are causes of secondary angina OR Type 2 MI (Type 1 Sontaneous)? a. An anemic patient from a gastrointestinal bleed b. A dialysis patient with an arterio-venous fistula c. A dyspneic patient with underlying emphysema d. a and c e. a, b, and c
  • 114. Answer to Q 18  Answer is e: Anemia, anterior-venous shunting, and hypoxemia can all cause demand ischemia. Note that a left-arm arterio-venous fistula can produce shunting as well as subclavian steal in patients with a previous left internal mammary artery graft.
  • 115. Q 19       Which of the following is not included in the differential diagnosis for electrocardiographic ST elevations? a. ST-elevation myocardial infarction b. Left ventricular aneurysm c. Hypokalemia d. Pericarditis e. Left ventricular hypertrophy
  • 116. Answer to Q 19  Answer is c: Among the electrolyte abnormalities, hyperkalemia, not hypokalemia can cause ST elevations that mimic ST-elevation myocardial infarctions.
  • 117. Q 20       Risk factors for intracranial hemorrhage during administration of fibrinolytics include all of the following except: a. Uncontrolled hypertension b. Advanced age c. Female gender d. Preexisting coagulopathy e. Morbid obesity
  • 118. Answer to Q 20  Answer is e: Low body weight, not morbid obesity, is a risk factor for intracranial hemorrhage.
  • 119.
  • 120.
  • 121.
  • 122.
  • 123.
  • 124. Q21       Digoxin toxicity may present with all the following except? a- Color visual defect b-GI upset c- Bidirectional VT d-headache e –all of the above
  • 125. Answer to Q 21  e –all of the above
  • 126.
  • 127. Q 22       Digoxin toxicity is more found except a-young men b- elederly females. c-Renal impairement. d-Liver impairment e- a,d
  • 129.
  • 130. Q23       All of the following clinical findings are consistent with severe mitral stenosis except A. atrial fibrillation B. opening snap late after S2 C. pulmonary vascular congestion D. pulsatile liver E. right-ventricular heave
  • 131. Answer to Q 23  B. opening snap late after S2
  • 132. Q 24       A patient is found to have a holosystolic murmur on physical examination.With deep inspiration, the intensity of the murmur increases.This is consistent with which of the following? A. Atrial-septal defect B. Austin Flint murmur C. Carvallo’s sign D. Chronic mitral regurgitation E. Gallavardin effect
  • 133. Answer to Q 24  C. Carvallo’s sign of TR  Carvallo's sign is a clinical sign found in patients with tricuspid regurgitation. The pansystolic murmur found in this condition becomes louder during inspiration this sign enables it to be distinguished from mitral regurgitation 
  • 134.
  • 136. 77 years old male with CHF,CRF
  • 137.
  • 138. Q 25       Which of the following findings would be suggestive of critical ischemia of the right foot? A. Ankle-brachial index <0.3 B. Ankle-brachial index <0.9 C. Ankle-brachial index >1.2 D. Lack of palpable dorsalis pedis pulse E. Presence of pitting edema of the extremities
  • 139. Answer to Q 25  A. Ankle-brachial index <0.3
  • 140. Q 26       All the following electrocardiogram (ECG) findings re suggestive of left ventricular hypertrophy except A. (S in V1 + R in V5 or V6) >35 mm B. R in aVL >11 mm C. R in aVF >20 mm D. (R in I + S in III) >25 mm E. R in aVR >8 mm
  • 141. Answer to Q 26  E. R in aVR >8 mm
  • 142. Q 27       55 years old saudi male , investigated for recent onset of exertional syncope , found to have AS , the most likely aetiology is ? A. Age-related degeneration B. Dyslipidemia C. Glucose intolerance D. Hypertension E. Obesity
  • 143. Answer to Q 27  A. Age-related degeneration
  • 144. Q 28      In peripartum cardiomyopathy Which of the following factors is the most valuable predictor of mortality with subsequent pregnancies? A. Age >30 years B. African ancestry C. Interpartum left ventricular function D. Male child
  • 145. Answer to Q 28  C. Interpartum left ventricular function
  • 146. 55 Years old lady with SOB & fatigability ECG & ECHO done
  • 147. 55 Years old lady with SOB & fatigability ECG & ECHO done NSR , Low Voltage ECG
  • 148.
  • 150.
  • 151.  Myxedema in Hypothyroidism  Atrial fibrillation might be the sole presentation of thyrotoxicosis. Atrial fibrillation may occur with Hypothyroidism Both Hypo-or Hyperthyroidism may cause Cardiomyopathy, wait for 3- 6 months before repeating ECHO after treament .  
  • 152. Q 29       HMG-CoA reductase inhibitor reduces coronary events. This Medication will exert all the following beneficial effects except? A. direct action on atheroma progression B. improvement in endothelial-dependent vasomotion C. long-term reduction of serum LDL D. regression of existing coronary stenosis E. stabilization of existing atherosclerotic lesions
  • 153. Answer to Q 29  D. regression of existing coronary stenosis
  • 154.
  • 155.
  • 156.  tendon xanthomas at the wrists, knees, and achilles, With ostial RCA
  • 157.
  • 159. Q 30    Which vein has the highest oxygen saturation? Which vein has the lowest oxygen saturation? In which situation is coronary sinus saturation elevated?
  • 160. Answer to Q 30  Renal vein has the highest oxygen saturation. Hence the inferior vena caval (IVC) blood is more saturated than superior vena caval blood. The saturation is higher in the inferior vena cava above the renal veins (high IVC) than the low IVC.
  • 161. Answer Q 30  Coronary sinus has the lowest oxygen saturation as the oxygen extraction is maximum in the coronary circulation. Coronary sinus saturation does not fall with exercise as the extraction reserve is fully utilized in the coronary circulation and increase in oxygen consumption can occur only by increasing the flow
  • 162. Q 31   Janeway lesion: a) Tender b) Nodular c) Most common in trunk d) Blanches on pressure
  • 163. A 31   Answer: d) Blanches on pressure Nodular and tender lesions are Osler’s nodes. Janeway lesions are seen on the palms and soles. They are non tender, macular lesions. These are lesions seen in infective endocarditis
  • 164. Q 32   HACEK organisms causing endocarditis includes: a) Cardiobacterium b) Acinetobacter c) E. coli d) Klebsiella
  • 165. A 32   Answer: a) Cardiobacterium HACEK is an acronym for a group of gram-negative bacilli: Haemophilus species (Haemophilus parainfluenzae, Haemophilus aphrophilus, Haemophilus paraphrophilus), Actinobacillus actinomycetemcomitans, Cardiobacterium hominis, Eikenella corrodens, and Kingella species. They have a higher potential to produce endocarditis and is responsible for about three percent of native valve endocarditis
  • 166. Q 33      Which of the following statements about infective endocarditis is wrong? a) Splenomegaly is more common in acute than sub acute infective endocarditis b) Disease manifestation occurs within two weeks of the cause of bacteremia c) Cerebral embolism is more common in the middle cerebral artery territory d) 5% chance of intracranial hemorrhage
  • 167. A 33  a) Splenomegaly is more common in acute than sub acute infective endocarditis
  • 168. Q34   Gene defective in Marfan syndrome: a) Collagen b) Fibrillin c) Elastin d) Cathepsin
  • 169. A 34   b) Fibrillin Marfan syndrome is caused by defects in a gene called fibrillin-1 (FBN1), located on chromosome 15.
  • 170. Q 35   For blood pressure measurement, least important is: a) Arm cuff at heart level b) Manometer at heart level c) Arm cuff >80% arm circumference d) Manometer should be vertical
  • 171. A 35  b) Manometer at heart level Manometer need not be at heart level as the difference involved will be only the weight of the air column in the tubing, which is negligible compared to that of mercury. But the mercury level should be at the level of your eye to avoid parallax error. A slanting mercury column will increase the reading. Position of arm cuff is important as level difference will alter the recorded pressure which will depend on the column of blood in the vessel above or below the heart level. Cuffs which are shorter will not provide adequate compression during inflation and is likely to record falsely elevated pressures
  • 172. Q 36   QT interval shortens in: a) Hypothermia b) Hypokalemia c) Hypocalcemia d) Acidosis
  • 173. A36   Answer: d) Acidosis Acidosis is associated with shift of potassium out of the cells and hyperkalemia. QT prolongation in hypocalcemia is due to ST segment prolongation. Digitalis and congenital short QT syndrome are other causes of a short QT interval.
  • 174. 60 years old male with CHF
  • 175. Q 37   According to BMI (body mass index), the cut off for obesity is: a) 25 Kg/m2 b) 30 Kg/m2 c) 35 Kg/m2 d) 40 Kg/m2
  • 176. A37       Answer: b) 30 Kg/m2 The International Classification of adult underweight, overweight and obesity according to BMI (World Health Organization) is as follows: Underweight <18.50 Normal range 18.50 – 24.99 Overweight ≥25.00 Pre-obese 25.00 – 29.99 Obese ≥30.00 Obese class I: 30.00 – 34.99 Obese class II: 35.00 – 39.99 Obese class III: ≥40.00
  • 177. Q 38
  • 178. A 38 Tamponade can be acute or subacute, depending on the etiology: Acute: Rapid accumulation (usually blood) within a stiff, noncompliant pericardium Subacute: Gradual increase of a preexisting effusion, with limited accommodative pericardial stretch
  • 179. Q 39
  • 180. A39
  • 181. Q 40
  • 182. A 40
  • 183. Q 41
  • 186. Q 42
  • 187. A 42  Dextrocardia Associated with situs inversus and kartageners syndrome.  Sometimes polysplenism 
  • 189. Q 43 ?????????? 25 years old with Flu
  • 190. A 43       Diffuse ST elevation with PR segment depression Inferior Leads.This is Acute Pericarditis. Treatment First Line A- High dose ASA 2- grams per day B- Paracetamol C-Cholchicine Avoid NSAIDs and steroids at least as first line , in post MI.
  • 191. Q 44 ?????? 60 male with CAD Where is the delta waves
  • 192.
  • 193. WPW syndrome with pseudo inferior MI pattern
  • 194. A 44       WPW Next step is Echo is needed because of increased incidence of associated structural heart disease with WPW. Treatment A- Asymptomatic Leave Alone B- Symptomatic offer EPS + ablation C- found and difficult to ablate in Ebstein anomaly
  • 195. Q 45
  • 196.  Atrial Flutter with 2:1 Block.
  • 197.
  • 198. B
  • 199.   25 year old female has a butterfly rash on her face, photosensitivity and oral ulcers. She is diagnosed with SLE. Which of the following is another characteristic of SLE? a) polyuria  b) polydipsia  c) polyphagia  d) pericarditis  e) Philadelphia chromosom
  • 201.   You are reviewing a patient in the intensive care unit with acute renal failure. Which antimicrobial agent is LEAST likely to require a dose adjustment in renal failure? a) gentamicin b) amitriptyline c) carbonic anhydrase inhibitors d) streptomycin e) erythromycin
  • 203.   A 44 year old woman with a long history of multiple sclerosis complains of severe pain on the right side of her face. The most likely pathology is in the a) the right facial nerve b) the right long thoracic nerve c) the right trigeminal nerve d) the left trigeminal nerve e) the right recurrent laryngeal nerve
  • 204. Correct Answer: C Pain typical of trigeminal neuralgia occasionally affects patients with lesions in the brain stem as a result of multiple sclerosis. They may also occur with vasculitis involving the descending root of the fifth cranial nerve. Trigeminal neuralgia usually occurs after other symptoms of MS. Of all patients with MS, however, about 10% have facial pain as a presentation, and other symptoms of MS may not appear for 6 yea 

Notas do Editor

  1. Multiple studies have validated the Duke criteria. When applied and reapplied over the entire evaluation, these criteria are sensitive and specific and very rarely erroneously reject a true endocarditis.